Теорема виета для неприведенного квадратного уравнения: формула, примеры, как решать, доказательство

Содержание

формула, примеры, как решать, доказательство

Основные понятия

Квадратное уравнение — это ax2 + bx + c = 0, где a — первый коэффициент, не равный нулю, b — второй коэффициент, c — свободный член.

Существует три вида квадратных уравнений:

  • не имеют корней;
  • имеют один корень;
  • имеют два различных корня.

Чтобы определить, сколько корней имеет уравнение, нужно обратить внимание на дискриминант. Формула для его поиска записывается так: D = b2 − 4ac. Его свойства:

  • если D < 0, корней нет;
  • если D = 0, есть один корень;
  • если D > 0, есть два различных корня.

В математике теоремой принято называть утверждение, у которого ранее было сформулировано доказательство.

Формула Виета



Если в школьной геометрии чаще всего используется теорема Пифагора, то в школьной алгебре ведущую роль занимают формулы Виета. Теорема звучит так: 

Теорема Виета

Сумма корней x2 + bx + c = 0 равна второму коэффициенту с противоположным знаком, а произведение корней равняется свободному члену.

Если дано x2 + bx + c = 0, где x₁ и x₂ являются корнями, то справедливы два равенства:

Знак системы, который принято обозначать фигурной скобкой, означает, что значения x₁ и x₂ удовлетворяют обоим равенствам.

Рассмотрим теорему Виета на примере: x2 + 4x + 3 = 0.

Пока неизвестно, какие корни имеет данное уравнение. Но в соответствии с теоремой можно записать, что сумма этих корней равна второму коэффициенту с противоположным знаком. Он равен четырем, значит будем использовать минус четыре:

Произведение корней по теореме соответствует свободному члену. В данном случае свободным членом является число три. Значит:

Необходимо проверить равна ли сумма корней −4, а произведение 3. Для этого найдем корни уравнения x2 + 4x + 3 = 0. Воспользуемся формулами для чётного второго коэффициента:

Получилось, что корнями уравнения являются числа −1 и −3. Их сумма равняется второму коэффициенту с противоположным знаком, а значит решение верное.

Произведение корней −1 и −3 по теореме Виета должно равняться свободному члену, то есть числу 3. Это условие также выполняется:

Результат проделанных вычислений в том, что мы убедились в справедливости выражения:


Доказательство теоремы Виета

Дано квадратное уравнение x2 + bx + c = 0. Если его дискриминант больше нуля, то оно имеет два корня, сумма которых равна второму коэффициенту с противоположным знаком, а произведение корней равно свободному члену:

Докажем, что следующие равенства верны

  • x₁ + x₂ = −b,
  • x₁ * x₂ = c.

Формулы корней

Чтобы найти сумму корней x₁ и x₂ подставим вместо них то, что соответствует им из правой части формул корней. Напомним, что в данном квадратном уравнении x2 + bx + c = 0 старший коэффициент равен единице. Значит после подстановки знаменатель будет равен 2.

  1. Объединим числитель и знаменатель в правой части.

     

  2. Раскроем скобки и приведем подобные члены:

     

  3. Сократим дробь полученную дробь на 2, остается −b:

     

Мы доказали: x₁ + x₂ = −b.

Далее произведем аналогичные действия, чтобы доказать о равенстве x₁ * x₂ свободному члену c.

  1. Подставим вместо x₁ и x₂ соответствующие части из формул корней квадратного уравнения:

     

  2. Перемножаем числители и знаменатели между собой:

     

  3. Очевидно, в числителе содержится произведение суммы и разности двух выражений. Поэтому воспользуемся тождеством (a + b) * (a − b) = a2 − b2. Получаем:

     

  4. Далее произведем трансформации в числителе:

     

  5. Нам известно, что D = b2 − 4ac. Подставим это выражение вместо D.

     

  6. Далее раскроем скобки и приведем подобные члены:

     

  7. Сократим:

     

Мы доказали: x₁ * x₂ = c.

Значит сумма корней приведённого квадратного уравнения x2 + bx + c = 0 равна второму коэффициенту с противоположным знаком (x₁ + x₂ = −b), а произведение корней равно свободному члену (x₁ * x₂= c). Теорема доказана.

Обратная теорема Виета

Когда дана сумма и произведение корней квадратного уравнения, принято начинать подбор подходящих корней. Теорема, обратная теореме Виета, при таких условиях может быть главным помощником. Она формулируется так:

Обратная теорема Виета

 

Если числа x₁ и x₂ таковы, что их сумма равна второму коэффициенту уравнения x2 + bx + c = 0, взятому с противоположным знаком, а их произведение равно свободному члену, то эти числа являются корнями x2 + bx + c = 0.

Обратные теоремы зачастую сформулированы так, что их утверждением является заключение первой теоремы. Так, при доказательстве теоремы Виета стало понятно, что сумма x₁ и x₂ равна −b, а их произведение равно c. В обратной теореме это является утверждением.

Докажем теорему, обратную теореме Виета

Корни x₁ и x₂ обозначим как m и n. Тогда утверждение будет звучать следующим образом: если сумма чисел m и n равна второму коэффициенту x

2 + bx + c = 0, взятому с противоположным знаком, а произведение равно свободному члену, то числа m и n являются корнями x2 + bx + c = 0.

Зафиксируем, что сумма m и n равна −b, а произведение равно c.

Чтобы доказать, что числа m и n являются корнями уравнения, нужно поочередно подставить буквы m и n вместо x, затем выполнить возможные тождественные преобразования. Если в результате преобразований левая часть станет равна нулю, то это будет означать, что числа m и n являются корнями x2 + bx + c = 0.

  1. Выразим b из равенства m + n = −b. Это можно сделать, умножив обе части на −1:

  2. Подставим m в уравнение вместо x, а выражение −m − n подставим вместо b:

При x = m получается верное равенство. Значит число m является искомым корнем.

  1. Аналогично докажем, что число n является корнем уравнения. Подставим вместо x букву n, а вместо c подставим m * n, поскольку c = m * n.
  2. При x = n получается верное равенство. Значит число n является искомым корнем.

Мы доказали: числа m и n являются корнями уравнения x2 + bx + c = 0.

Примеры

Для закрепления знаний рассмотрим примеры решения уравнений по теореме, обратной теореме Виета.

Дано: x2 − 6x + 8 = 0.

Для начала запишем сумму и произведение корней уравнения. Сумма будет равна 6, так как второй коэффициент равен −6. А произведение корней равно 8.

Имея эти два равенства можно подобрать подходящие корни, которые будут удовлетворять как равенству обоим равенствам системы.

Подбор корней удобнее выполнять с помощью их произведения. Число 8 можно получить путем перемножения чисел 4 и 2 либо 1 и 8. Но значения x₁ и x₂ надо подбирать так, чтобы они удовлетворяли и второму равенству тоже.

Можно сделать вывод, что значения 1 и 8 не подходят, так как они не удовлетворяют равенству x₁ + x₂ = 6. Значения 4 и 2 подходят обоим равенствам:

Значит числа 4 и 2 являются корнями уравнения x2 − 6x + 8 = 0.

Неприведенное квадратное уравнение 

Теорема Виета выполняется только тогда, когда квадратное уравнение является приведённым, то есть его первый коэффициент равен единице:

ax2 + bx + c = 0, где а = 1.

Если квадратное уравнение не является приведенным, но задание связано с применением теоремы, нужно обе части разделить на коэффициент, который располагается перед x2.

  1. Получилось следующее приведенное уравнение:

  1. Получается коэффициент равен , свободный член — . Значит сумма и произведение корней будут иметь вид:

  2. Рассмотрим пример неприведенного уравнения: 4x2 + 5x + 1 = 0. Разделим обе его части на коэффициент перед x2, то есть на 4.

  3. Получилось приведённое квадратное уравнение. Второй коэффициент которого равен , а свободный член .
  4. Тогда в соответствии с теоремой Виета получаем:

  5. Метод подбора помогает найти корни: −1 и 


 



Теорема Виета

Предварительные навыки

Что называют теоремой?

Если человек обнаружил в математике какую-нибудь закономерность, позволяющую быстро решить ту или иную задачу, то ему не следует говорить о том, что он сделал открытие. Потому что может случиться так, что эта закономерность работает только для определённых случаев, а для других не работает или вовсе решает задачу неправильно.

Чтобы поделиться своим открытием с другими людьми, найденную закономерность следует сформулировать в виде утверждения, а затем доказать это утверждение, приводя неоспоримые факты.

Сформулированное утверждение называют теоремой. А доказательство теоремы состоит из фактов, логических рассуждений и вычислений, которые не оспариваются.

Например, теоремой можно назвать следующее утверждение:

«Если числитель и знаменатель обыкновенной дроби умнóжить на какое-нибудь число, то значение данной дроби не измéнится».

А затем привести такое доказательство:

Пусть, имеется дробь . Умнóжим числитель и знаменатель этой дроби на число с. Тогда полýчится дробь . Докáжем, что дроби  и равны. То есть докажем, что равенство является верным.

Для доказательства этого равенства воспользуемся основным свойством пропорции:

От перестановки мест сомножителей произведение не меняется. Поэтому в получившемся равенстве можно упорядочить правую часть по алфавиту:

Поскольку равенство является пропорцией, а пропорция это равенство двух отношений, то дроби и равны. Теорема доказана.


Теорема Виета

Французский математик Франсуа Виет выявил интересную взаимосвязь между коэффициентами приведённого квадратного уравнения и корнями этого же уравнения. Эта взаимосвязь представлена в виде теоремы и формулируется так:

Сумма корней приведённого квадратного уравнения xbx = 0 равна второму коэффициенту, взятому с противоположным знáком, а произведение корней равно свободному члену.

То есть, если имеется приведённое квадратное уравнение xbx = 0, а его корнями являются числа x1 и x2, то справедливы следующие два равенства:

Знак системы (фигурная скобка) говорит о том, что значения x1 и x2 удовлетворяют обоим равенствам.

Покажем теорему Виета на примере приведённого квадратного уравнения x+ 4+ 3 = 0.

Мы пока не знаем какие корни имеет уравнение x+ 4+ 3 = 0. Но по теореме Виета можно записать, что сумма этих корней равна второму коэффициенту 4, взятому с противоположным знáком. Если коэффициент 4 взять с противоположным знáком, то получим −4. Тогда:

А произведение корней по теореме Виета будет равно свободному члену. В уравнении x+ 4+ 3 = 0 свободным членом является 3. Тогда:

Теперь проверим действительно ли сумма корней равна −4, и равно ли произведение 3. Для этого найдём корни уравнения x+ 4+ 3 = 0. А для удобства воспользуемся формулами для чётного второго коэффициента:

Корнями уравнения являются числа −1 и −3. По теореме Виета их сумма должна была равняться второму коэффициенту уравнения x+ 4+ 3 = 0, взятому с противоположным знаком. Действительно, так оно и есть. Вторым коэффициентов в уравнении x+ 4+ 3 = 0 является 4. Если взять его с противоположным знаком и приравнять сумму корней xx2 к этому коэффициенту, то получается верное равенство:

А произведение корней −1 и −3 по теореме Виета должно было равняться свободному члену уравнения x+ 4+ 3 = 0, то есть числу 3. Видим, что это условие тоже выполняется:

Значит выражение  является справедливым.


Рассмотрим квадратное уравнение x− 8+ 15 = 0. По теореме Виета сумма корней этого уравнения равна второму коэффициенту, взятому с противоположным знаком. Второй коэффициент равен −8. Если взять его с противоположным знаком, то получим 8. Тогда:

А произведение корней равно свободному члену. В уравнении x− 8+ 15 = 0 свободным членом является 15. Тогда:

Теперь проверим действительно ли сумма корней равна 8, и равно ли произведение 15. Для этого найдём корни данного уравнения. А для удобства воспользуемся формулами для чётного второго коэффициента. В этот раз пропустим нéкоторые подробные записи:

Видим, что корнями уравнения x− 8+ 15 = 0 являются числа 5 и 3. Их сумма равна 8. То есть сумма корней равна второму коэффициенту уравнения x− 8+ 15 = 0, взятому с противоположным знаком.

А произведение чисел 5 и 3 равно 15. То есть равно свободному члену уравнения x− 8+ 15 = 0.

Значит выражение является справедливым.

Замечание. Чтобы теорема Виета выполнялась, квадратное уравнение обязательно должно быть приведённым и иметь корни.

Например, рассмотрим квадратное уравнение x− 2+ 4 = 0. Напишем сумму и произведение корней этого уравнения:

Но уравнение x− 2+ 4 = 0 не имеет корней, сумма которых равна 2, а произведение которых равно 4. Убедиться в этом можно, вычислив дискриминант:

D1 = k− ac = (−1)− 1 × 4 = −3

А значит записывать выражение не имеет смысла.

Теорема Виета полезна тем, что позволяет до начала решения узнать знаки корней уравнения.

Например, запишем для уравнения x− 5+ 6 = 0 сумму и произведение его корней. Сумма корней равна второму коэффициенту, взятому с противоположным знаком, а произведение корней равно свободному члену:

Посмотрев на эти два равенства можно сразу понять, что оба корня должны быть положительными. Потому что произведение x× x= 6 будет выполняться только в двух случаях: если значения x1 и x2 положительны либо они оба отрицательны. Если эти значения будут отрицательными, то не будет выполняться равенство xx= 5, поскольку его правая часть равна положительному числу. А значения x1 и x2 должны удовлетворять как равенству xx= 5, так и равенству x× x= 6.

Ещё одна польза от теоремы Виета в том, что корни можно найти методом подбора. В данном примере корни должны быть такими, чтобы они удовлетворяли как равенству xx= 5 так и равенству x× x= 6. Очевидно, что таковыми являются корни 3 и 2

Значит, x= 3, x= 2


Доказательство теоремы Виета

Пусть дано приведённое квадратное уравнение xbx = 0. Если его дискриминант больше нуля, то оно имеет два корня, сумма которых равна второму коэффициенту, взятому с противоположным знаком, а произведение корней равно свободному члену:

Докажем, что равенства xx= −b и x× xc имеют место быть.

Вспомним формулы корней квадратного уравнения:

Найдём сумму корней x1 и x2. Для этого подставим в выражение xx2 вместо x1 и x2 соответствующие выражения из правой части формул корней квадратного уравнения. Не забываем, что в приведённом квадратном уравнении xbx = 0 старший коэффициент a равен единице. Тогда в процессе подстановки знаменатель станет равен просто 2

Запишем правую часть в виде дроби с одним знаменателем:

Раскроем скобки в числителе и приведём подобные члены:

Сократим дробь на 2, тогда получим −b

Значит xx2 действительно равно −b

xx= −b

Теперь аналогично докажем, что произведение x× x2 равно свободному члену c.

Подставим вместо x1 и x2 соответствующие выражения из формул корней квадратного уравнения. Не забываем, что коэффициент a всё ещё равен единице:

Чтобы перемнóжить дроби, нужно перемнóжить их числители и знаменатели:

В числителе теперь содержится произведение суммы двух выражений и разности этих же выражений. Воспользуемся тождеством (a + b)(a − b) = a− b2. Тогда в числителе полýчится А знаменатель будет равен 4

Теперь в числителе выражение (−b)2 станет равно b2, а выражение станет равно просто D

Но D равно b− 4ac. Подстáвим это выражение вместо D, не забывая что = 1. То есть вместо b− 4ac надо подставить b− 4c

В получившемся выражении раскроем скобки в числителе и приведём подобные члены:

Сократим получившуюся дробь на 4

Значит x× x2 действительно равно c.

x× xc

Таким образом, сумма корней приведённого квадратного уравнения xbx = 0 равна второму коэффициенту, взятому с противоположным знáком (xx= −b), а произведение корней равно свободному члену (x× xc). Теорема доказана.


Теорема, обратная теореме Виета

Когда записана сумма и произведение корней приведённого квадратного уравнения, обычно начинается подбор подходящих корней к этому уравнению. В этот момент в работу включается так называемая теорема, обратная теореме Виета. Она формулируется так:

Если числа x1 и x2 таковы, что их сумма равна второму коэффициенту уравнения xbx = 0, взятому с противоположным знáком, а произведение чисел x1 и x2 равно свободному члену уравнения xbx = 0, то числа x1 и x2 являются корнями уравнения xbx = 0.

Обратные теоремы бывают поставлены так, что их утверждением является заключение первой теоремы.

Так, доказывая теорему Виета мы пришли к заключению, что сумма x1 и x2 равна −b, а произведение x1 и x2 равно c. В обратной же теореме это заключение служит утверждением.

Ранее мы решили уравнение x− 5+ 6 = 0 и написали для него такую сумму и произведение корней:

А затем подобрали корни 3 и 2. По сути мы применили теорему, обратную теореме Виета. Числа 3 и 2 таковы, что их сумма равна второму коэффициенту уравнения x− 5+ 6 = 0, взятому с противоположным знаком (числу 5), а произведение чисел 3 и 2 равно свободному члену (числу 6). Значит числа 3 и 2 являются корнями уравнения x− 5+ 6 = 0.


Пример 2. Решить квадратное уравнение x− 6+ 8 = 0 по теореме, обратной теореме Виета.

В данном уравнении = 1. Значит квадратное уравнение является приведённым. Его можно решить по теореме, обратной теореме Виета.

Сначала запишем сумму и произведение корней уравнения. Сумма корней будет равна 6, поскольку второй коэффициент исходного уравнения равен −6. А произведение корней будет равно 8

Теперь имея эти два равенства можно подобрать подходящие корни. Они должны удовлетворять как равенству xx= 6, так и равенству x× x= 8

Подбор корней удобнее выполнять с помощью их произведения. Используя равенство x× x= 8 нужно найти такие x1 и x2, произведение которых равно 8.

Число 8 можно получить если перемножить числа 4 и 2 либо 1 и 8.

4 × 2 = 8
1 × 8 = 8

Но значения x1 и x2 надо подбирать так, чтобы они удовлетворяли не только равенству x× x= 8, но и равенству xx= 6.

Сразу делаем вывод, что значения 1 и 8 не годятся, поскольку они хоть и удовлетворяют равенству x× x= 8, но не удовлетворяют равенству xx= 6.

Зато значения 4 и 2 подходят как равенству x× x= 8, так и равенству xx= 6, поскольку эти значения удовлетворяют обоим равенствам:

Значит корнями уравнения x− 6+ 8 = 0 являются числа 4 и 2.

Обратная теорема, как и любая теорема нуждается в доказательстве. Докажем теорему, обратную теореме Виета. Для удобства корни x1 и x2 обозначим как m и n. Тогда утверждение теоремы, обратной теореме Виета примет следующий вид:

Если числа m и n таковы, что их сумма равна второму коэффициенту уравнения xbx = 0, взятому с противоположным знáком, а произведение чисел m и n равно свободному члену уравнения xbx = 0, то числа m и n являются корнями уравнения xbx = 0

Для начала запишем, что сумма m и n равна −b, а произведение mn равно c

Чтобы доказать, что числа m и n являются корнями уравнения xbx = 0, нужно поочередно подстáвить буквы m и n в это уравнение вместо x, затем выполнить возможные тождественные преобразования. Если в результате преобразований левая часть станет равна нулю, то это будет означать, что числа m и n являются корнями уравнения xbx = 0.

Помимо букв m и n нам нужно знать чему равен параметр b. Выразим его из равенства m + n = −b. Легче всего это сделать, умножив обе части этого равенства на −1

Теперь всё готово для подстановок. Подстáвим m в уравнение xbx = 0 вместо x, а выражение −m − n подставим вместо b

Видим, что при x = m получается верное равенство. Значит число m является корнем уравнения xbx = 0.

Аналогично докажем, что число n является корнем уравнения xbx = 0. Подставим вместо x букву n, а вместо c подставим mn, поскольку c = mn.

Видим, что при x = n тоже получается верное равенство. Значит число n является корнем уравнения.

Следовательно, числа m и n являются корнями уравнения xbx = 0.


Примеры решения уравнений по теореме, обратной теореме Виета

Пример 1. Решить квадратное уравнение x− 4+ 4 = 0 по теореме, обратной теореме Виета.

Запишем сумму корней x1 и x2 и приравняем её к второму коэффициенту, взятому с противоположным знаком. Также запишем произведение корней x1 и x2 и приравняем его к свободному члену:

В данном примере очевидно, что корнями являются числа 2 и 2. Потому что их сумма равна 4 и произведение равно 4

Значение x1 совпадает с x2. Это тот случай, когда квадратное уравнение имеет только один корень. Если мы попробуем решить данное уравнение с помощью формул корней квадратного уравнения, то обнаружим что дискриминант равен нулю, и корень вычисляется по формуле

Данный пример показывает, что теорема обратная теореме Виета, работает и для уравнений, имеющих только один корень. Признаком того, что квадратное уравнение имеет только один корень является то, что значения x1 и x2 совпадают.


Пример 2. Решить уравнение x+ 3+ 2 = 0 по теореме, обратной теореме Виета.

Запишем сумму и произведение корней данного уравнения:

Теперь подберём значения x1 и x2. Здесь начинается самое интересное. Произведение корней равно 2. Число 2 можно получить перемножив 1 и 2. Но сумма корней xx2 равна отрицательному числу −3. Значит значения 1 и 2 не подходят.

Сумма бывает отрицательной если оба слагаемых отрицательны либо отрицательным является одно слагаемое, модуль которого больше.

Если подберём корни с разными знаками, то не будет выполняться равенство x× x= 2.

Если подберем положительные корни, то будет выполняться равенство x× x= 2, но не будет выполняться равенство xx= −3.

Очевидно, что корнями являются два отрицательных числа. Произведение отрицательных чисел есть положительное число. А сумма отрицательных чисел есть отрицательное число.

Тогда равенствам будут удовлетворять числа −1 и −2.

Итак, корнями являются числа −1 и −2


Пример 3. Решить уравнение x+ 16+ 15 = 0 по теореме, обратной теореме Виета.

Запишем сумму и произведение корней данного уравнения:

Как и в прошлом примере сумма корней равна отрицательному числу, а произведение корней — положительному числу.

Произведение бывает положительным если оба сомножителя положительны либо оба сомножителя отрицательны. Первый вариант отпадает сразу, поскольку сумма корней равна отрицательному числу. Тогда получается, что оба корня будут отрицательными. Попробуем подобрать их.

Число 15 можно получить, если перемножить числа −1 и −15 или (−3) и (−5). В данном случае подходит первый вариант, поскольку сумма чисел −1 и −15 равна −16, а их произведение равно 15. Значит корнями уравнения x+ 16+ 15 = 0 являются числа −1 и −15


Пример 4. Решить уравнение x− 10− 39 = 0 по теореме, обратной теореме Виета.

Запишем сумму и произведение корней данного уравнения:

Произведение корней равно отрицательному числу. Значит один из корней является отрицательным. Число −39 можно получить если перемножить числа −3 и 13 либо −13 и 3. Из этих комбинаций больше годится комбинация −3 и 13, поскольку при перемножении этих чисел получается −39, а при сложении 10

Значит корнями уравнения x− 10− 39 = 0 являются числа −3 и 13


Пример 5. Первый корень уравнения xbx + 45 = 0 равен 15. Найти второй корень этого уравнения, а также значение коэффициента b.

По теореме Виета произведение корней приведённого квадратного уравнения равно свободному члену. В данном случае это произведение равно 45

x1 × x2 = 45

При этом один из корней уже известен — это корень 15.

15 × x2 = 45

Тогда второй корень будет равен 3, потому что число 45 получается, если 15 умножить на 3

15 × 3 = 45

Значит x2 = 3

Этот второй корень также можно было бы получить, выразив из равенства 15 × x2 = 45 переменную x2

Теперь определим значение коэффициента b. Для этого напишем сумму корней уравнения:

15 + 3 = 18

По теореме Виета сумма корней приведенного квадратного уравнения равна второму коэффициенту, взятому с противоположным знаком. Если сумма корней равна 18, а 18 это положительное число, то в самóм уравнении этот коэффициент будет отрицательным:

x2 − 18+ 45 = 0

Значит = −18.

Обычно решение к такой задаче записывают так. Сначала записывают основную теорему Виета в виде суммы и произведения корней:

Затем в это выражение подставляют имеющиеся известные значения. В нашем случае известно, что первый корень равен 15, а свободный член уравнения xbx + 45 = 0 равен 45

Из этой системы следует найти x2 и b. Выразим эти параметры:

Из этой системы мы видим, что x2 равно 3. Подставим его в первое равенство:

Теперь из первого равенства мы видим, что −b равно 18

Но нас интересует b, а не −b. Следует помнить, что −b это −1b. Чтобы найти b нужно 18 разделить на −1. Тогда b станет равно −18

Этот же результат можно получить если в выражении умножить первое равенство на −1

Теперь возвращаемся к исходному уравнению xbx + 45 = 0 и подставляем найденное значение b

Выполним умножение −18 на x. Получим −18x

Раскроем скобки:


Пример 6. Используя теорему Виета, написать приведённое квадратное уравнение, корнями которых являются числа 2 и 8.

В этом задании корни уже известны. То есть x= 2, x= 8. По ним надо составить квадратное уравнение вида xbx = 0.

Запишем сумму и произведение корней:

По теореме Виета сумма корней приведённого квадратного уравнения равна второму коэффициенту, взятому с противоположным знаком. Если сумма корней 2 и 8 равна 10, то в самóм уравнении число 10 должно быть с противоположным знаком. Значит = −10.

Произведение корней по теореме Виета равно свободному члену. У нас это произведение равно 16.

Значит = −10, = 16. Отсюда:

x2 − 10+ 16 = 0


Пример 7. Используя теорему Виета, написать приведённое квадратное уравнение, корнями которых являются числа и .

Запишем сумму и произведение корней:

Сумма корней равна 2. Тогда в уравнении второй коэффициент будет равен −2. А произведение корней равно −1. Значит свободный член будет равен −1. Тогда:

x2 − 2x − 1 = 0


Когда квадратное уравнение неприведённое

Теорема Виета выполняется только тогда, когда квадратное уравнение является приведённым.

Если квадратное уравнение не является приведённым, но всё равно возникла необходимость применить теорему Виета, то обе части неприведённого квадратного уравнения следует разделить на коэффициент, который располагается перед x2.

Если к примеру в квадратном уравнении axbx = 0 коэффициент a не равен единице, то данное уравнение является неприведённым. Чтобы сделать его приведённым, надо разделить обе его части на коэффициент, который располагается перед x2, то есть на a

Получилось уравнение , которое является приведённым. В нём второй коэффициент равен , а свободный член равен . Тогда сумма и произведение корней будут выглядеть так:

Например, решим квадратное уравнение 4x+ 5+ 1 = 0. Это уравнение не является приведённым. Приведённым оно станет, если разделить обе его части на коэффициент, который располагается перед x2, то есть на 4

Получили приведённое квадратное уравнение. В нём второй коэффициент равен , а свободный член . Тогда по теореме Виета имеем:

Отсюда методом подбора находим корни −1 и

Возможно этот метод вы редко будете использовать при решении квадратных уравнений. Но знать о нём не помешает.


 

Пример 2. Решить квадратное уравнение 3x− 7+ 2 = 0

Данное уравнение не является приведённым, а значит его пока нельзя решить по теореме, обратной теореме Виета.

Сделаем данное уравнение приведенным. Разделим обе части на коэффициент, который располагается перед x2

Получили уравнение . Запишем сумму и произведение корней этого уравнения:

Отсюда методом подбора находим корни 2 и


Пример 3. Решить квадратное уравнение 2x− 3− 2 = 0

Это неприведённое квадратное уравнение. Чтобы сделать его приведённым, нужно разделить обе его части на 2. Сделать это можно в уме. Если 2x2 разделить на 2, то полýчится x2

Далее если −3x разделить на 2, то полýчится . Чтобы видеть где коэффициент, а где переменная, такое выражение записывают в виде

Далее если −2 разделить на 2, то полýчится −1

Прирáвниваем получившееся выражение к нулю:

Теперь применяем теорему Виета. Сумма корней будет равна второму коэффициенту, взятому с противоположным знáком, а произведение корней свободному члену:

Отсюда методом подбора находим корни 2 и


Задания для самостоятельного решения

Задание 1. Написать сумму и произведение корней для квадратного уравнения:

Решение:

Задание 2. Написать сумму и произведение корней для квадратного уравнения:

Решение:

Задание 3. Написать сумму и произведение корней для квадратного уравнения:

Решение:

Задание 4. Решить квадратное уравнение по теореме, обратной теореме Виета:

Решение:

Задание 5. Решить квадратное уравнение по теореме, обратной теореме Виета:

Решение:

Задание 6. Решить квадратное уравнение по теореме, обратной теореме Виета:

Решение:

Задание 7. Решить квадратное уравнение по теореме, обратной теореме Виета:

Решение:

Задание 8. Решить квадратное уравнение по теореме, обратной теореме Виета:

Решение:

Задание 9. Решить квадратное уравнение по теореме, обратной теореме Виета:

Решение:


Понравился урок?
Вступай в нашу новую группу Вконтакте и начни получать уведомления о новых уроках

Возникло желание поддержать проект?
Используй кнопку ниже

Навигация по записям

Теорема Виета для кубического уравнения

1. Теорема Виета для кубического уравнения.

Муниципальное образовательное учреждение
«Средняя общеобразовательная школа №44»
Теорема Виета для
кубического
уравнения.
Работа выполнена учениками 11 «А» класса
Емельяновым Тимофеем и
Вдовенковой Алёной
Крамаренко Елена Андреевна,
учитель математики
высшей категории
г. Саратов
2016 год

2. Историческая справка

Виет Франсуа родился в
1540 году в Фонте-ле-Конт
французской провинции
Пуату – Шарант. Отец
Виета был юристом
(прокурором), а мать
(Маргарита Дюпон)
происходила из знатной
семьи, что облегчило
дальнейшую карьеру её
сына.
Получив юридическое образование, он с
девятнадцати лет успешно занимался
адвокатской практикой в родном городе. Он был
широко образованным человеком. Знал
астрономию и математику и все свободное
время отдавал этим наукам.
Но главной страстью Виета была математика.
Виет сделал принципиально новое открытие, поставив
перед собой цель изучать не числа, а действия над
ними. Виета называют «отцом» алгебры,
основоположником буквенной символики. Особенно
гордился Франсуа всем известной теперь теоремой о
выражении коэффициентов уравнения через его корни.
Виет первый обозначил буквами не только неизвестные, но
и данные величины, т.е. коэффициенты
соответствующих уравнений
Виет сначала решает задачи в общем виде, и
только потом приводит числовые
параметры. В общей части он обозначает
буквами не только неизвестные, что уже
встречалось ранее, но и все прочие
параметры, для которых он придумал
термин «коэффициенты». Виет использовал
для этого только заглавные буквы – гласные
для неизвестных, согласные для
коэффициентов.

6. Теорема Виета

Сумма корней приведённого квадратного уравнения x2 + px + q = 0
равна коэффициенту p, взятому с противоположным знаком, а
произведение корней равно свободному члену q:
х1 х2 p
х1 х2 q
В общем случае (для неприведенного квадратного уравнения):
в
х1 х 2
а
с
х1 х2
а
Особый интерес
представляет
исследование Виета
по составлению
уравнений из
линейных
множителей и по
установлению связей
между корнями
уравнения и его
коэффициентами.
Пусть x1 и x2 – корни приведенного квадратного
уравнения x2 + px + q = 0
Перемножим двучлены (х — x1) и (х — x2) :
(х — x1)(х — x2) = x2 — (x1+ x2)х + x1x2 ,
Тогда, сравнивая с исходным уравнением
можно записать систему :
p ( x1 x2 )
q x1 x2
Выполняя аналогичные действия для приведенного
кубического уравнения x3+ax2+bx+c=0, считая x1,x2,x3
корнями исходного кубического уравнения, получаем:
(х-x1)(х-x2)(х-x3) = x3 – (x1+x2+x3 ) x2+(x1x2+ x1x3 + x2,x3)х
— x1x2x3
следовательно, имеет место следующая система
равенств:
а ( х1 х2 х3 ),
в х1 х2 х1 х3 х2 х3 ,
с х х х
1 2 3
Если
x1,x2,x3,- корни
неприведённого кубического уравнения
ax3 + bx2 + cx + d = 0, то
x1+x2+x3=- b/a
(х1*х2+х1*х3+х2*х3)= с/а
х1*х2*х3=-d/a
есть суть теоремы Виета для кубического
уравнения.

11. Решить уравнение x3-4×2+x+6=0.

1 способ: при а=1 свободный член этого уравнения раскладывают на
простые множители, затем поочередно выбирают значения «x»,
равные одному из этих множителей с различными знаками. Эти
значения х проверяют, подставляя их в исходное равенство. Таким
способом иногда удается найти первый корень кубического уравнения
х1. Для нахождения остальных корней кубического уравнения надо
соответствующий многочлен разделить на выражение (х-х1), при этом
в частном получается квадратный трехчлен. Корни получившегося
квадратного трехчлена также являются корнями кубического
уравнения. Таким образом 6=1*2*3, т.е.корни уравнения могут быть
числа 1 или -1,2 или-2,3 или-3. Способом подстановки выясняем, что
х1=-1.Разделим многочлен х3-4х2+х+6 на (х+1) и получим трехчлен
х2-5х+6, т.е.х3-4х2+х+6=(х+1)*(х2-5х+6)=0.
Найдем корни квадратного уравнения х2-5х+6=0 по теореме Виета:
х1=2 и х2=3.
Таким образом исходное кубическое уравнение имеет три
действительных корня:
х1=-1, х2=2, х3=3.
2 способ: применение теоремы Виета для решения
кубического уравнения
Итак, если х3-4х2+х+6=0,
то х1+х2+х3=4
Х1*х2+х1*х3+х2*х3=1,
Х1*х2*х3=-6
Методом подбора находим: (-1)*2*3=-6
(-1)+2+3=4
(-1)*2+(-1)*3+2*3=1,
т.е корни уравнения
х1=-1,х2=2, х3=3.

13. Задача: вычислить, используя теорему Виета, сумму квадратов корней уравнения х3-6х2+11х-6=0

Согласно теореме Виета имеем:
х1+х2+х3=6
х1*х2+х1*х3+х2*х3=11
х1*х2*х3=6
Т.к. (х1+х2+х3)2=х12+х22+х32+2(х1*х2+х1*х3+х2*х3)
то получим 62=х12+х22+х32+2*11
36-22= х12+х22+х32
х12+х22+х32=14
Ответ: 14

14. Задача: Составить кубическое уравнение, корнями которого являются числа -5;3;4

Решение:
Пусть х1=-5, х2=3,х3=4, тогда
а ( 5 3 4),
в 5 * 3 ( 5) * 4 3 * 4,
с ( 5) * 3 * 4
а 2,
в 23,
с 60;
А теперь составим приведенное кубическое уравнение
вида x3+ax2+bx+c=0, корнями которого являются -5,
3, 4. Им будет x3-2×2-23x+60=0

15. Посвящение теореме Виета:

По праву достойна в стихах быть воспета
О свойствах корней теорема Виета.
Что лучше, скажи постоянства такого:
Умножишь ты корни — и дробь уж готова:
В числителе с, в знаменателе а,
А сумма корней тоже дроби равна
Хоть с минусом дробь эта, что за беда
В числителе в, в знаменателе а.

Тема: ТЕОРЕМА ВИЕТА 8 Класс

Тема: ТЕОРЕМА ВИЕТА

Тип урока: изучение нового материала.

Класс: 8.

Цель урока: а) Знакомство с теоремой Виета и следствием.

б) Применение теоремы Виета в различных ситуациях.

в) развивать логическое мышление через мыслительные операции умение делать выводы, анализировать, применять знания в нестандартных ситуациях.

Материалы и оборудование урока: компьютор, презентация.

Ход урока (урок сопровождается презентацией):

I. Постановка цели урока.

На прошлых уроках вы познакомились с новыми уравнениями.

  • Назовите их и дайте определение.

  • В зависимости от наличия коэффициентов, на какие группы делятся квадратные уравнения?

  • В зависимости от значения коэффициента а на какие группы делятся квадратные уравнения?

  • Дайте определение приведенного квадратного уравнения.

Также познакомились с формулами корней квадратного уравнения и дискриминантом.

  • Какую связь устанавливают формулы корней квадратного уравнения?

  • Какую зависимость устанавливает значение дискриминанта?

А как вы думаете: все ли связи между корнями и коэффициентами квадратного уравнения мы рассмотрели?

Откройте тетради, запишите число и тему урока.

II. Проверка домашнего задания и формулирование проблемы.

1

Исследование связи между корнями

и коэффициентами квадратного уравнения.

Обратимся к домашней работе. Дома вы решали 4 уравнения и заполняли таблицу.

2.

Проверьте свою работу по таблице. А теперь посмотрите внимательно, что интересного вы заметили? Обсудите это в парах и попытайтесь сформулировать предположение (выслушиваем предположения). Действительно, вы правы, существует такое утверждение.

III. Изучение нового материала.

3.

Утверждение №1:

Пусть х1и х2 – корни уравнения х2+pх+q=0.

Тогда числа х1, х2 , p, q связаны равенствами:

х12= -p, х1х2=q

Утверждение № 2:

Пусть числа х12,p,q связаны равенствами х12= -p, х1х2=q.

Тогда х1и х2 – корни уравнения х2+pх+q=0

Записываем в тетрадях и доказываем данные утверждения. (в процессе доказательства обратного утверждения получается равенство: х2+рх+q=(х-х1)(х-х2)).

Итак, мы доказали теорему Виета. Запишите ее в тетрадях.

4

Теорема Виета:

Числа х1 и х2 являются корнями приведенного квадратного уравнения х2+pх+q=0 тогда и только тогда, когда х12= -p, х1х2=q.

Следствие: х2+pх+q=(х-х1)(х-х2).

Как вы догадались, что данная теорема носит имя автора. Кем же был Франсуа Виет, и когда была доказана эта теорема?

№5
Франсуа Виет

Франсуа Виет родился в 1540 году во Франции. Отец Виета был прокурором. Сын выбрал профессию отца и стал юристом, окончив университет в Пуату. В 1563 году он оставляет юриспруденцию и становится учителем в знатной семье. Именно преподавание побудило в молодом юристе интерес к математике.

Виет переезжает в Париж, где легче узнать о достижениях ведущих математиков Европы. С 1571 года Виет занимает важные государственные посты, но в 1584 году он был отстранен и выслан из Парижа. Теперь он имел возможность всерьез заняться математикой.

В 1591 году он издает трактат «Введение в аналитическое искусство», где показал, что, оперируя с символами, можно получить результат, применимый к любым соответствующим величинам. Знаменитая теорема была обнародована в том же году.

Громкую славу получил при Генрихе lll во время Франко-Испанской войны. В течение двух недель, просидев за работой дни и ночи, он нашел ключ к Испанскому шифру.

Умер в Париже в 1603 году, есть подозрения, что он был убит.

Итак, какие связи между коэффициентами и корнями приведенного квадратного уравнения мы обнаружили? Чем интересно полученное следствие? Где это можно использовать?

Подумайте и ответьте: где, в каких ситуациях можно воспользоваться теоремой и следствием?

Свои предположения обсудите в парах и полученные ситуации запишите в тетрадь.

(выслушиваем то, что получилось, обсуждаем)

Давайте сравним ваши предположения с предлагаемыми ситуациями.

6Ситуации, в которых может использоваться теорема Виета.

  • Проверка правильности найденных корней.

  • Определение знаков корней квадратного уравнения.

  • Устное нахождение целых корней приведенного квадратного уравнения.

  • Составление квадратных уравнений с заданными корнями.

  • Разложение квадратного трехчлена на множители.

IV. Самостоятельная работа учащихся.

Выполним задания.

7

Решите следующие задания:

  1. Верно ли, что числа 15 и 7 являются корнями уравнения х2 -22х+105=0?

  2. Определите знаки корней уравнения х2+5х-36=0.

  3. Найдите устно корни уравнения х2 -9х+20=0.

  4. Составьте квадратное уравнение, корнями которого являются числа 1/3 и 0,3.

  5. Разложите квадратный трехчлен на множители х2+2х-48.

V. Постановка новой проблемы

Используя ответ задания №4, сначала переходим к квадратному уравнению с целыми коэффициентами, а затем задается вопрос: будет ли верна теорема Виета для данного неприведенного квадратного уравнения?

Учащиеся в парах обсуждают возникшую проблему, пробуют сформулировать по аналогии обобщенную теорему Виета. Обсуждаем полученные варианты ответов. Затем выясняем, как бы выглядело следствие для таких уравнений. Сравниваем со следующим слайдом.

8

Обобщенная теорема Виета:

Числа х1 и х2 являются корнями квадратного уравнения ах2+bх+с=0 тогда и только тогда, когда х12= -b/а, х1х2=с/а.

Следствие: ах2+bх+c=а(х-х1)(х-х2).

VI. Применение полученных знаний.

Учащимся предлагается применить полученные знания в следующих ситуациях.

9

Решите следующие задания:

  1. В уравнении х2+pх-32=0 один из корней равен 7. Найдите другой корень и коэффициент p.

  2. Один из корней уравнения 10х2 -33х+с=0 равен 5,3. Найдите другой корень и коэффициент с.

  3. Разность корней квадратного уравнения х2 -12х+q=0 равна 2. Найдите q.

  4. Определите знаки корней квадратного уравнения ( если они существуют), не решая уравнения: 5х2-х- 108=0.

  5. Найдите b и решите уравнение (b-1)· х2— (b+1)х = 72, если х1 = 3.

Учащиеся решают в парах, полученные решения объясняют у доски. После решения этих заданий подводится итог урока.

VII. Итог урока. Задание на дом.

10

Итог урока:

Домашнее задание

11

Домашнее задание:

Корни уравнения являются натуральными числами. Доказать, что — составное число.

п.24.прочитать, выучить теоремы и следствия.

582(а, б, в), №584(а), №596(д)

Теорема Виета Пусть х1 и х2

Теорема Виета

Пусть х1 и х2 – корни уравнения х2+pх+q=0. Тогда числа х1, х2 , p, q связаны равенствами: х1+х2= -p, х1 х2=q

Дано: х2 + рх + q = 0 приведённое квадратное уравнение, x 1, x 2 – корни уравнения Доказать: x 1+ x 2=-p x 1 x 2=q

Доказательство: • Чему равен дискриминант уравнения и определите знак дискриминанта? • Запишите корни уравнения:

Найдите сумму и произведения корней: Итак, мы доказали теорему Виета. Запишите ее в тетрадях.

Теорема Виета для приведенного квадратного уравнения: x 2 +bx + c= 0 x 1 + x 2 = -b x 1 * x 2 = c –Сумма корней приведенного квадратного уравнения равна второму коэффициенту, взятому с противоположным знаком, а произведение корней равно свободному члену.

Теорема Виета для неприведенного квадратного уравнения: (обобщенная теорема Виета) b a x 1 +x 2 = ax 2 + bx +c = 0 x 1 x 2 = c a Следствие: ах2+bх+c = а(х-х1)(х-х2).

По праву в стихах быть достойна воспета О свойствах корней теорема Виета. Скажи, что может быть лучше постоянства такого, Умножишь ты корни и дробь уж готова В числителе с, в знаменателе а. А сумма корней тоже дроби равна, Хоть с минусом дробь — это что за беда, В числителе в, в знаменателе а.

Утверждение, обратное теореме Виета Пусть числа х1, х2, p, q связаны равенствами х1+х2= -p, х1 х2=q. Тогда х1 и х2 – корни уравнения х2+pх+q=0. Следствие: х2+pх+q=(х-х1)(х-х2).

Франсуа Виет • Франсуа Виет родился в 1540 году во Франции. Отец Виета был прокурором. Сын выбрал профессию отца и стал юристом, окончив университет в Пуату. В 1563 году он оставляет юриспруденцию и становится учителем в знатной семье. Именно преподавание побудило в молодом юристе интерес к математике. • Виет переезжает в Париж, где легче узнать о достижениях ведущих математиков Европы. С 1571 года Виет занимает важные государственные посты, но в 1584 году он был отстранен и выслан из Парижа. Теперь он имел возможность всерьез заняться математикой. • В 1591 году он издает трактат «Введение в аналитическое искусство» , где показал, что, оперируя с символами, можно получить результат, применимый к любым соответствующим величинам. Знаменитая теорема была обнародована в том же году. • Громкую славу получил при Генрихе lll во время Франко-Испанской войны. В течение двух недель, просидев за работой дни и ночи, он нашел ключ к Испанскому шифру. • Умер в Париже в 1603 году, есть подозрения, что он был убит.

Ситуации, в которых может использоваться теорема Виета. • Проверка правильности найденных корней. • Определение знаков корней квадратного уравнения. • Устное нахождение целых корней приведенного квадратного уравнения. • Составление квадратных уравнений с заданными корнями. • Разложение квадратного трехчлена на множители.

• • • Решите следующие задания: Верно ли, что числа 15 и 7 являются корнями уравнения х2 -22 х+105=0? Определите знаки корней уравнения х2+5 х-36=0. Найдите устно корни уравнения х2 -9 х+20=0. Составьте квадратное уравнение, корнями которого являются числа 1/3 и 0, 3. Разложите квадратный трехчлен на множители х2+2 х-48.

Технологическая карта урока по теме : Теорема Виета

Урок по теме « Теорема Виета» 8 класс.

Тип урока : комбинированный.

Используемые технологии: дифференцированное обучение , проблемное обучение.

Формы учебной деятельности учащихся: индивидуальная , групповая.

Оборудование: мультимедийный проектор.

Продолжительность урока 35 мин.

План урока.

  1. Организационный момент (1 мин)

  2. Проверка домашнего задания (1 мин)

  3. Устный опрос (5мин)

  4. Выполнение задания исследовательского характера (7 мин)

  5. Объяснение нового материала (3 мин)

  6. Закрепление учебного материала(10 мин)

  7. Тестирование (5 мин)

  8. Задание на дом (1мин)

  9. Рефлексия (1 мин).

Ход урока.

Цель: изучить теорему Виета.

Задачи:

  • Обучить умению решать приведенные квадратные уравнения с помощью теоремы Виета;

  • Развивать внимание, логическое и математическое мышление, умение анализировать;

  • Воспитывать интерес математике.

  1. Организационный момент:

Учитель объявляет тему, цели и план урока.

  1. Наличие домашнего задания проверяется до урока. К тем номерам , которые вызвали затруднения дать пояснения на уроке ( или пригласить на кружок по математике).

  2. Устная работа:

В ходе устной работы учитель должен определить на сколько хорошо учащиеся владеют основными понятиями о квадратных уравнениях, какие пробелы есть в знаниях?

Фронтальный опрос способствует развитию коммуникативных навыков общения и говорения, способствует воспитанию у учащихся взаимопомощи.

  1. Дайте определение полных, неполных и приведенных квадратных уравнений.

  2. Назовите полные, неполные и приведенные квадратные уравнения:

2 – 2х = 0 -21х2 + 16х=0

2 – 16х + 4 =0 х2=0

Х2 – 3 = 0 х2 + 4х + 4 =0

— х2 +2х — 4 =0 х2=4

  1. Преобразуйте квадратное уравнение в приведенное:

2 + 6х – 12 =0 0,2х2 + 0,5х – 2 =0

2 – 2х + 16 = 0 -5х2 + 10х -2 =0

4. От чего зависит наличие или отсутствие корней квадратного уравнения?

5. По какой формуле находятся корни квадратного уравнения?

4) «Открытие» теоремы Виета.

Организовываем исследовательскую деятельность, что позволяет активизировать учебно -познавательную деятельность учащихся и повысить интерес к предмету.

Для этого разбиваем класс на пять групп, каждой из которых дается задание решить приведенное уравнение. После его решения один представитель от каждой группы выходит к доске и заполняет соответствующую строку в таблице:

Уравнение

b

c

Корни

Сумма корней

Произведение корней

Х2 +11х -12 = 0

Х2 – 6х – 7 = 0

Х2 + 3х – 10 = 0

Х2+ 5х + 6 = 0

Х2– х – 12 = 0

После этого учитель предлагает учащимся сравнить сумму и произведение полученных корней с коэффициентами b и c и выдвинуть гипотезу. Учитель подтверждает сделанное предположение, сообщая, что данное утверждение называется теоремой Виета, обращая внимание учащихся, что эта теорема справедлива для приведенных квадратных уравнений.

Краткий исторический материал:

Француа Виет – французский математик, ввел систему алгебраических символов, разработал основы элементарной алгебры. Он был одним из первых кто стал обозначать числа буквами, что существенно развило теорию уравнений.

Рассмотреть доказательство теоремы можно по учебнику (с. 127– 128), привлекая учащихся, поскольку оно не является сложным. После доказательства на доску выносится запись:

Т е о р е м а В и е т а

Если х1, х2 – корни уравнения x2 + px + q = 0,

то х1 + х2 = –р; х1 · х2 = q.

Для первичного усвоения теоремы Виета можно предложить учащимся выполнить устно упражнение на нахождение суммы и произведения корней квадратного уравнения:

1) х2 – 37х + 27 = 0

2) х2х – 5 = 0.

3) х2 + 3х + 5 = 0.

При выполнении этого задания необходимо предотвратить формальное применение теоремы Виета. Нужно убедиться, что квадратное уравнение имеет корни. Если учащиеся сами не выскажут эту мысль, то при решении третьего задания предложить им найти дискриминант уравнения и сделать соответствующий вывод.

Далее рассматривается вывод корней неприведенного квадратного уравнения.

При выполнении устной работы в начале урока учащиеся вспомнили, как преобразовать квадратное уравнение в приведённое. Следует предложить им самостоятельно вывести формулы для неприведённого квадратного уравнения, используя теорему Виета. После этого на доску выносится запись:

Т е о р е м а В и е т а

Если х1, х2 – корни уравнения аx2 + bx + c = 0,

то х1 + х2 = ; х1х2 = .

Чосер – английский поэт средних веков, сказал:

« Посредством уравнений, теорем,

Я уйму разрешил проблем».

Выучив теорему Виета , вы тоже разрешите для себя уйму всяких проблем.

  1. На данном этапе учитель на доске показывает решение приведенного квадратного уравнения с использованием теоремы Виета. Акцентирует внимание учащихся на перебор корней, знаки, быстроту решения.

№ 583

х2-9х + 20=0 Д0

х1х2=20=4* 5= -4 * (-5)= 2 * 10= -2 * (-10) = 1*20=-1 * (-20)

из этих множителей выбираем х1=4 , х2= 5 так как х1 + х2= 9.

  1. Закрепление учебного материала.

Учащиеся выполняют на доске №583, №587, №588.

Все допущенные ошибки разбираются всем классом.

№ 589 разбирается учителем на доске.

Устно:

Составьте квадратное уравнение, имеющее следующие корни:

X1

X2

X1•X2

X1+X2

Уравнение

2

5

 

 

 

2

 

0,8

 

 

 

 

8

-6

 

 

 

8

6

 

4

-3

 

 

 

12

0,5

 

 

 

С целью определения степени усвоения учащимися нового материала дается тест с тремя уровнями сложности с учетом индивидуальных особенностей учащихся.

  1. Тест (листы) на 3 (1-3 задания)

  1. Укажите в квадратном уравнении х2+3 -4х =0 второй коэффициент:

1 -4 3 4

  1. В квадратном уравнении 7х – 5 – х2 =0 второй коэффициент с противоположным знаком равен?

-1 1 5 -7

3. Сумма и произведение корней уравнения х2 +7х – 1=0 равны:

х1 + х2 = 7; х1 · х2 = 1. х1 + х2 = 1; х1 · х2 = 7.

х1 + х2 = -7; х1 · х2 = -1. х1 + х2 = –1; х1 · х2 = -7.

на 4 (1-5 задание)

4.Если число 11 корень уравнения х2 – 13х +22 =0, то второй корень равен:

13 -11 2 -2

5.Если 2 корень уравнения х2 – 6х +q =0, то q равен

12 8 -12 6

на 5 ( 1-7задания)

6. Не решая уравнение х2-9х-4=0, определите знаки корней уравнения:

одинаковые оба положительные

разные оба отрицательные

7. Для уравнения -9х2+2х -4 =0 приведенным является уравнение вида:

Х2+ х — =0 Х2+ х — =0

Х2 + 2х -4 =0 -х2-2х =4 =0

№ задания

1

2

3

4

5

6

7

ответ

С помощью теста так же вырабатываются навыки выполнения работ в тестовой форме необходимые на ГИА и проводится рефлексия (дорисуй настроение) «смайлик»

— урок заставляет задуматься;

— урок был обычен;

— урок был интересен.

8) Домашнее задание по двум уровням сложности. Ученики самостоятельно выбирают уровень сложности домашнего задания. Но учащимся претендующим на оценки «4» и «5» рекомендуется выбрать домашнее задание второго уровня сложности. По желанию учащихся предлагается написать доклад «Француа Виет»

I №584, 585,541 (е,ж)

II № 582, 587,591

  1. Подведение итогов занятия:

  2. На данном этапе дается анализ и оценка успешности достигнутых целей урока. По результатам урока выставляются оценки. Намечаются перспективы последующей работы.

Решение задач по теме «Теорема Виета» | План-конспект урока по алгебре (8 класс) на тему:

Тема: Решение задач по теме «Теорема Виета».

8 класс.
Учитель Лукина И.В.

На уроке используется технология проблемного обучения.

Цели урока:

а) Образовательные:
— обеспечение прочности приобретенных знаний, умений и навыков по теме «Теорема Виета»;

— исследование различных подходов к решению нестандартных задач.

б) Развивающие:
— развитие самостоятельности, ответственности;
— развитие критичности мышления, нестандартного мышления;
— развитие творческих способностей учащихся.

в) Воспитательные:

— воспитание творческой личности;

— воспитание коммуникативных качеств ученика.

Задача: изучить новые способы решения полных квадратных уравнений. Для этого  организовать на уроке проблемные ситуации, оказать учащимся помощь в решении этих проблем, проверить решения.

Ход урока.
Учащиеся работают в четырех разноуровневых группах.

  1. Вводная часть

Ученик читает стихотворение, подготовленное заранее:
По праву достойна в стихах быть воспета
О свойствах корней теорема Виета.
Что лучше, скажи, постоянства такого:
Умножишь ты корни – и дробь уж готова?
В числителе с, в знаменателе а.
А сумма корней тоже дроби равна.
Хоть с минусом дробь эта, что за беда!
В числителе b, в знаменателе а.

  1. Проверка творческого домашнего задания.
     На дом было задано сочинить стихотворение, небольшой рассказ или сказку по теме «Теорема Виета». Это задание было задано на неделю. Работы были собраны заранее. Трое учащихся, чьи работы признаны лучшими, выступают перед классом.
  2. Объявляется тема урока. 
    Ставится задача: изучить новые нестандартные способы решения полных квадратных уравнений.
  3. Постановка 1проблемы.
    Теорема Виета не зря воспета в стихах. Она очень облегчает решение приведенных квадратных уравнений, но мы никогда не использовали ее при решении неприведенных квадратных уравнений. А возможно ли это в принципе?
    Проведем эксперимент. Для этого рассмотрим уравнение 3х2 – 8х + 5 = 0. Его можно решить по формуле, которая применяется для решения полных квадратных уравнений.

                Х1 = 1; Х2 = 5/3

Рассмотрим теперь приведенное квадратное уравнение, которое можно получить из  данного неприведенного, для этого мы свободный член умножим на старший коэффициент.

                         
                         Х2 – 8Х + 5·3 = 0

                          Х2 – 8Х + 15 = 0
Это уравнение легко решается по теореме, обратной теореме Виета.
Обозначим эти корни Х3 и Х4

                Х3 = 3; Х4 = 5.
Сравнивая корни уравнений, получаем, что корни второго уравнения в три раза больше корней первого, т.е. Х1 = Х3/3 и Х2 = Х4/3
Возникает проблема: всегда ли корни неприведенного квадратного уравнения
ax2 + bx + c = 0 и корни приведенного квадратного уравнения x2 + bx + c·a = 0, полученного из первого умножением старшего коэффициента на свободный член, cвязаны равенством   Х1 = Х3/а и Х2 = Х4/а, где Х1 и Х2 – корни неприведенного уравнения, а Х3 и Х4 – корни приведенного уравнения. Было бы здорово решить эту проблему сегодня на уроке. А вы как думаете, это случайное совпадение или нет?
          5. Поиск решения проблемы и признание решения учащимися. 

Учащимся дается возможность высказаться, посовещавшись с членами группы и доказать или опровергнуть выдвинутую гипотезу. Если дети затрудняются, то учитель предлагает подсказки по очереди, давая после каждой подсказки время для обдумывания дальнейшего хода решения и высказывания каждой группе. Подсказки предлагаются на слайдах презентации:
1 подсказка. 
Проверьте еще раз зависимость между корнями уравнений 7Х2 – 6Х — 1 = 0 и
Х2 – 6Х —  1·7 = 0

Двое учащихся работают на досках, а остальные – в тетрадях.

1)7Х2 – 6Х — 1 = 0

          6 ± √62 — 4·7·1

Х1,2 =


Х1 = 1; Х2 = — 1/7

2)Х2 – 6Х —  1·7 = 0

Х2 – 6Х —  7 = 0 – по теореме, обратной теореме Виета находим корни этого уравнения:

Х3 = 7; Х4 = — 1.

Класс делает вывод:  Х1 = Х3/7 и Х2 = Х4/7.
Таким образом, Х1 = Х3/а и Х2 = Х4/а.

2 подсказка. 
Попробуйте записать оба уравнения: неприведенное и полученное из него приведенное квадратные уравнения в общем виде и применить к каждому из них формулу для решения полных квадратных уравнений.
Учащиеся работают в группах, и если они затрудняются, то учитель им помогает.
Представители от групп выходят к доске и объясняют решение. На доске появляются записи:

1) а Х2 + bХ +  c = 0

 

          — b ± √b2 — 4·a·c

Х1,2 =

2) Х2 + bХ +  c·a = 0

 

          — b ± √b2 — 4·a·c

Х3,4 =

3 подсказка. 
Сравните правые части равенств (1) и (2).
Учащиеся снова работают в группах. А затем представители от групп выходят к доске и объясняют решение.
Из формул видно, что если правую часть равенства (2) разделить на а, то получится правая часть равенства (1).
Разделим обе части равенства (2) на а (напоминаю, что на а делить можем т.к. а ≠ 0 по определению квадратного уравнения).

Запись на доске:

         

 

              — b ± √b2 — 4·a·c

Х3,4 =  

Сравнивая полученное равенство с равенством (1) получаем, что правые части этих равенств равны, следовательно, равны и левые. Таким образом
Х1 = Х3/а и Х2 = Х4/а.
Доказательство учащиеся, по мере обсуждения, записывают в тетради.

  1. Обобщение и выводы.
    Мы доказали, что неприведенные квадратные уравнения можно решать не только по общей формуле, а так же по теореме, обратной теореме Виета.

Алгоритм решения:
1) составить приведенное квадратное уравнение из данного неприведенного, умножив свободный член на старший коэффициент.
2) решить полученное уравнение по теореме, обратной теореме Виета.
3) получить корни заданного уравнения, разделив корни полученного  приведенного квадратного уравнения на старший коэффициент.

Алгоритм записывается учащимися в тетради.

  1. Упражнения.

Каждой группе дается задание на карточке:
1) решить данное учителем уравнение по записанному алгоритму
2) составить и решить свое уравнение для другой группы (творческое задание).
1)Уравнения, данные учителем для групп:
1 группе: 2Х2 – 5Х — 3 = 0;
Запись в тетрадях: Х2 – 5Х — 3·2 = 0;
                                Х2 – 5Х — 6 = 0;
                                Х3 = — 1; Х4 = 6;

                                 Х1 = — 1/2; Х2 = 3.


2 группе: 2Х2 –Х — 6 = 0;
Запись в тетрадях: Х2 – Х — 6·2 = 0;
                                Х2 – Х — 12 = 0;
                                Х3 = — 3; Х4 = 4;

                                 Х1 = — 1.5; Х2 = 2.


3 группе: 6Х2 –Х — 1 = 0;
Запись в тетрадях: Х2 – Х — 6·1 = 0;
                                Х2 – Х — 6 = 0;
                                Х3 = — 2; Х4 = 3;

                                 Х1 = — 1/3; Х2 = 1/2.


4 группе: 5Х2 + 2Х — 3 = 0;
Запись в тетрадях: Х2 + 2Х — 3·5 = 0;
                                Х2 + Х — 15 = 0;
                                Х3 = — 5; Х4 = 3;

                                 Х1 = — 1; Х2 = 3/5.


Учащиеся в группе обсуждают решение, записывают его по алгоритму, и представители от каждой группы выходят к доске и объясняют решение. Остальные слушают.
2) Для составления квадратных уравнений необходимо рассуждать точно в обратном порядке, а именно: сначала по корням, используя теорему Виета составить приведенное квадратное уравнение, а далее, разложив на множители свободный член, получить неприведенное уравнение.  
Далее каждая группа решает уравнение, предложенное другой группой: 1 группа составляет уравнение для 2 группы, 2 группа – для 3 группы, 3 группа – для 4 группы, 4 группа – для 1 группы. Решения оформляются на листах и передаются в группу, которая это уравнение составила, для проверки и выставления оценки за правильность решения и оформление. После чего листы с решением и отметками сдаются учителю.

  1. Постановка 2 проблемы.

Дано квадратное уравнение с большими коэффициентами:
1978Х2 – 1984Х + 6 = 0. Решить его по общей формуле не просто, слишком большие коэффициенты. Теорема Виета то же не решит проблему т.к. подобрать корни в уравнении с такими большими коэффициентами очень долговременно. И вообще, имеет ли это уравнение корни, и если – да, то как их найти рациональным способом?

  1. Поиск решения проблемы и признание решения учащимися.

Учащимся дается возможность высказаться, посовещавшись с членами группы, и попытаться решить поставленную проблему. Если дети затрудняются, то учитель снова дает подсказки по очереди, давая после каждой подсказки время для обдумывания дальнейшего хода решения и высказывания, используя слайды презентации:

1 подсказка.
 Найдите сумму коэффициентов уравнения.
Сумма коэффициентов равна нулю (1978 – 1984 + 6 = 0).
2 подсказка.
 Исходя из того, что сумма коэффициентов равна нулю, подбором найдите один корень уравнения.

Х1 = 1.

3 подсказка.
 Получите изданного уравнения приведенное, разделив обе части его на старший коэффициент 1978.

1978Х2 – 1984Х + 6 = 0
Х2 – 1984/1978Х + 6/1978 = 0.

4 подсказка. 
По теореме, обратной теореме Виета, найдите второй корень уравнения.

Х1·Х2 = 6/1978
Х1 = 1, следовательно 1·Х2 = 6/1978, значит Х2 = 6/1978 или Х2 = 3/989
Ответ: 1; 3/989.
Решение учащиеся записывают в тетради.

  1. Выводы.
    Этот урок можно назвать творческим т.к. на нем были заслушаны лучшие детские сочинения и решались квадратные уравнения нестандартными способоми. Для этого вы пытались самостоятельно выявить связи между уравнениями, учились переносить «старые» знания на новые ситуации и задачи, приучались к самоконтролю, самопроверке и взаимопроверке.
  2. Итоги урока.
    Благодарность всем учащимся за работу.
    Каждый ученик получит оценку за урок, которая будет складываться из оценки за решение проблем (активное обсуждение и выступление), за работу на доске, за решение уравнений, предложенных учителем в группе и оценки на листах, выставленной учащимися. Оценки будут объявлены на следующем уроке после суммирования.
  3. Домашнее задание (творческое).
    1) придумать 5 уравнений с большими коэффициентами и решить их;
    2) № 434 – решить уравнения по изученному на уроке алгоритму, используя теорему, обратную теореме Виета;
    3) для уравнения 3х2 – 8х + 5 = 0 найти х12 + х22, не решая уравнение.
     

предварительное вычисление алгебры — Есть ли общая формула для решения уравнений 4-й степени (квартики)?

Конечно, есть, но это некрасиво, сложно и не стоит запоминать. Люди знают об этом и цитировали или цитировали это для вас, но на самом деле они никогда бы не использовали это. Если вам нужно что-то действительно полезное для решений с ручкой и бумагой, вы можете понять фактическую теорию, лежащую в основе решения. Ниже я приведу один метод.

Формула Quartic — это лишь конечный результат этой методологии, записанный в терминах исходных коэффициентов.Из-за этого метод намного легче запомнить, чем формулу, поэтому меня раздражает, когда люди цитируют только формулу и говорят вам: «Не беспокойтесь, используйте компьютер». Решение с ручкой и бумагой не сложно, это просто требует времени.

Понимание того, как это делается, даже если вы никогда его не используете, расширяет ваш мозг и ваше понимание, позволяет реализовать его в программировании и позволяет воссоздавать его всякий раз, когда это может вам понадобиться, вместо чрезмерной зависимости от компьютеров, которые всегда будут рядом. вы, что, на мой взгляд, плохой математик.

Есть три метода решения квартиков, которые я знаю и знаю:

  • Квадратичная факторизация Декарта
  • Метод Эйлера
  • Метод Феррари

Если кто-нибудь знает больше, дайте мне знать.

Метод Феррари — исторически первый метод, открытый. Метод Эйлера очень похож на метод Кардано для кубических форм и, вероятно, был смоделирован на основе того же подхода. Но я неравнодушен к технике квадратичной факторизации Декарта.Это относительно простой процесс, который я буду использовать ниже. Если вы хотите увидеть, как работают другие, дайте мне знать.

Все вышеперечисленные методы начинаются одинаково: депрессия (удаление члена степени $ n-1 $, в данном случае кубического члена) и нормализация (доведение коэффициента опережения до 1, т. Е. Превращение полинома в однозначное).

Они все заканчивают примерно одним и тем же местом: решением кубического уравнения. Так что к этому нужно быть готовым. Я рекомендую вам освежить в памяти это; Я не буду объяснять здесь решение кубики, а буду ссылаться только на нее.2 + qz + r = 0 \; \; \; \; \; \; \; \; \; \; \; \; (1) $$

Для некоторых $ p, q, r \ in \ mathbb {R} $. Это не влияет на корневые позиции; все, что сделал коэффициент опережения, — увеличил ненулевые значения. Абсолютно без потери общности по нулям. Все эти константы, $ p, q, r $, могут быть вычислены из исходных коэффициентов, $ a, b, c, d, e $. Кубического члена по-прежнему нет, и теперь нет и коэффициента опережения.

Интересно отметить, что случилось с многочленом.Мы начали с 5 произвольных констант и сократили их до 3, нормализовав шаг и убрав кубический член. Изначально у нас были произвольные значения $ a, b, c, d, e \ in \ mathbb {R} $, а теперь у нас есть произвольные значения $ p, q, r \ in \ mathbb {R} $. Хотя последние три вычисляются из исходных пяти, они имеют произвольные значения и нет потери общности. Это значительное упрощение проблемы. Отсутствие кубического члена окажется жизненно важным.

До сих пор все было просто настройкой: запись многочлена в сокращенной монической форме.Вспомните, что все методы четвертой степени достигают по крайней мере этого. Далее мы реализуем метод факторизации Декарта.


Метод факторизации Декарта

Мы должны предполагать, что все коэффициенты действительны, $ p, q, r \ in \ mathbb {R} $. 2 } $$

Таким образом, мы, по сути, сократились до $ m $ как последнее неизвестное.2 = 0 $$

И, по сути, мы закончили. Остается кубический многочлен от $ w $ , который разрешим с помощью собственных методов. Приемы, о которых, как я полагаю, вы уже знаете, если пытаетесь решить квартики. Как и в случае с квартиками, как вы уже знаете, существуют кубические формулы, но я рекомендую изучить методы, лежащие в их основе.

Если вам нужна помощь с кубиками, я рекомендую метод Кардано (исходное решение) или тригонометрическое решение Виета (я предпочитаю).Также есть Completing the Cube, хорошее доказательство концепции, но я бы никогда его не использовал. Смело задавайте по кубику отдельный вопрос и я с радостью отвечу.

Дело в том, что задача свелась с поиска корней квартики к задаче поиска корней кубики. Проблема попроще! Обычно так бывает. Все методы нахождения корня четвертой степени требуют сначала нахождения корней кубической, очевидной или нет. Так же, как поиск корней кубики требует решения квадратичной.2 — mz + \ frac {r} {n}) = 0 $.

Еще не сделано. Теперь каждый из этих квадратичных множителей должен быть решен с помощью формулы квадратичного уравнения, и у вас есть решения в $ z $. Это решает депрессивную моническую квартику, с которой мы начали метод квадратичной факторизации Декарта.


Наконец

Не забывайте об исходной квартике, которая была у нас в самом начале, до депрессии и нормализации. Мы ввели горизонтальный сдвиг на $ x = z- \ frac {b} {4a} $. Выполнение этого последнего бита решит исходную квартику в терминах $ x $, что и является решением, которое вы хотите.

Когда все будет готово, вы придете к набору решений. Обязательно проверьте свои ответы. У вас могут быть избыточные или лишние решения. Некоторые избыточные решения могут быть записаны очень разными алгебраическими способами, но будут представлять одно и то же числовое значение.

Если вы выразите окончательный ответ $ x $ в терминах исходных $ a, b, c, d, e $, вы получите точно такие же «формулы четвертой степени», которые другие люди цитируют. Выражение, конечно, будет немного отличаться в зависимости от того, какой из методов четвертой степени вы используете.


Проблемы

Если вас беспокоит предположение, что коэффициенты $ p, q, r $ действительны, не стоит. Все это означает, что $ a, b, c, d, e $ реальны, что обычно является хорошим предположением. Фактически, мы можем обобщить. Значения $ p, q, r $ можно сделать сложными, подразумевая только то, что исходная квартика имеет комплекс $ a, b, c, d, e $. Это также означает, что вам придется решать кубику с комплексными коэффициентами. Это выполнимо, и математика по-прежнему работает нормально.

связанных слов — поиск слов, связанных с другим словом

Как вы, наверное, заметили, слова, относящиеся к слову «термин», перечислены выше.Надеюсь, сгенерированный список слов, связанных с терминами, соответствует вашим потребностям.

П.С. Есть некоторые проблемы, о которых я знаю, но в настоящее время не могу их исправить (потому что они выходят за рамки этого проекта). Главный из них заключается в том, что отдельные слова могут иметь много разных значений (значений), поэтому, когда вы ищете такое слово, как означает , движок не знает, к какому определению вы имеете в виду («хулиганы означают » vs . «что вы означает ?» и т. д.), поэтому учтите, что ваш поисковый запрос для таких слов, как термин, может быть немного неоднозначным для движка в этом смысле, и соответствующие термины, которые возвращаются, могут отражать это.Вам также может быть интересно: что за слово ~ термин ~?

Также проверьте слова ~ term ~ на relatedwords.io, чтобы найти еще один источник ассоциаций.

Связанные слова

Related Words работает по нескольким различным алгоритмам, которые соревнуются за повышение своих результатов в списке. Один из таких алгоритмов использует встраивание слов для преобразования слов в многомерные векторы, которые представляют их значения. Векторы слов в вашем запросе сравниваются с огромной базой данных предварительно вычисленных векторов, чтобы найти похожие слова.Другой алгоритм просматривает Concept Net в поисках слов, которые имеют какое-то значимое отношение к вашему запросу. Эти и некоторые другие алгоритмы позволяют «Родственным словам» давать вам … связанных слов, а не просто прямые синонимы.

Помимо поиска слов, связанных с другими словами, вы можете вводить фразы, и он должен давать вам связанные слова и фразы, если введенная фраза / предложение не слишком длинное. Вы, вероятно, время от времени будете получать какие-то странные результаты — это просто природа движка в его текущем состоянии.

Особая благодарность разработчикам кода с открытым исходным кодом, который был использован для предоставления вам этого списка тематических слов: @Planeshifter, @HubSpot, Concept Net, WordNet и @mongodb.

Еще предстоит проделать большую работу, чтобы добиться стабильно хороших результатов, но я думаю, что это на той стадии, когда это может быть полезно для людей, поэтому я выпустил его.

Обратите внимание, что «Связанные слова» используют сторонние скрипты (такие как Google Analytics и рекламные объявления), которые используют файлы cookie.Чтобы узнать больше, см. Политику конфиденциальности.

Zekeriya Kaya (математический сайт — изучайте математику с помощью видео)

ГЛАВА 1 ЧИСЛОВЫЕ СИСТЕМЫ

1) ЕСТЕСТВЕННЫЕ ЧИСЛА

— Смешанные операции в натуральных числах Видео 1, Видео 2, Видео 3, Видео 4, Видео 5 , Видео 6, Видео 7, Видео 8, Видео 9, Видео 10, Видео 11, Видео 12, с 12:30 до 24:00 ВИДЕО 90 ,

— Вычислить сумму последовательных натуральных чисел Видео 1, Видео 2, Видео 3, Видео 4, Видео 5, ВИДЕО 6 , с 24:30 до конца ВИДЕО 90 , с 00:00 до 17:00 ВИДЕО 91 ,

— Проблема О сумме последовательных натуральных / нечетных / четных чисел

— Дополнительные занятия с общими натуральными числами с 6-м классом ВИДЕО 90 , ВИДЕО 91 ,

ГЛАВА 2 МОДУЛЬНАЯ АРИФМЕТИКА

1) АРИФМЕТИЧЕСКИЕ И МОДУЛЬНЫЕ ЧАСЫ

9

902) ОПЕРАЦИИ В МОДУЛЬНОЙ АРИФМЕТИКЕ

3) БИНАРНЫЕ ОПЕРАЦИИ

ГЛАВА 3 ЭКСПОНЕНТЫ И РАДИКАЛЫ

1) РЕАЛЬНЫЕ НОМЕРА

2) АБСОЛЮТНЫЕ ЧИСЛА С ЧИСЛОМ 9011

9

— Экспоненциальные уравнения

4) КВАДРАТНЫЕ КОРНИ

-Свойства квадратных корней

-Простые квадратные корни

-Умножение и деление квадратных корней

-Сложение и вычитание

-Сравнение квадратных корней Видео 1,

-Квадратный корень из иррациональной суммы / A Частный случай для квадратных корней Видео 1,

5) РАЦИОНАЛЬНЫЕ ПОКАЗАТЕЛИ

6) РАДИКАЛЫ

7) РАДИКАЛЬНЫЕ УРАВНЕНИЯ

.

ГЛАВА 4 НАБОРЫ

1) КОНЦЕПЦИЯ НАБОРОВ Видео 1,

a) Наборы

b) Представляющие наборы Vid 1 , Vid 2 ,

c) Количество элементов

d) Типы наборов

e) Подмножества

— Подмножества, включающие или не включающие отдельные элементы Vid 1 ,

— Подмножества с определенным количеством элементов Vid 1 ,

Число подмножеств по треугольнику Паскаля

2) ОПЕРАЦИИ НАБОРЫ

a) Объединение наборов

b) Пересечение наборов

c) Дополнение набора

d) Разница из двух наборов

3) ПРИЛОЖЕНИЯ НА НАБОРЫ (Использование свойств набора для решения письменных задач)

-ПРОБЛЕМЫ ИССЛЕДОВАНИЯ о наборах uni на, пересечение, разность, дополнение Вид 1 , Вид 2 , Вид 3 ,

Количество элементов объединения двух наборов

Количество элементы объединения трех наборов

4) МАТЕМАТИЧЕСКИЕ КОЛИЧЕСТВЕННИКИ

Универсальный квантификатор; » для всех, для каждого »

Экзистенциальный квантор; » для некоторых хотя бы один »

.

.

ГЛАВА 5 ФУНКЦИИ

1) ОТНОШЕНИЯ

Упорядоченные пары

Декартово произведение

Аналитическая плоскость

Взаимосвязи

Определение функций

2) ВВЕДЕНИЕ

2) ВВЕДЕНИЕ a Функция

— Результаты поиска для кусочно определенной функции Vid 1 ,

Типы путем проверки домена и диапазона

Один к одному — В — В

Рисование графика функций первой степени

Домен функций

Четные нечетные функции

Возрастающие, убывающие и постоянные функции

Чтение графика функций Видео 1,

3) ОПЕРАЦИИ НАД ФУНКЦИЯМИ

Базовые операции

Состав функций Vid28, Видео 1, Видео 2, Видео 3,

Инверсия функций Vid 1 , Video 1,

— Поиск результатов для инверсии функции Vid 1 ,

4) ГРАФИК ФУНКЦИЙ (опционально)

Преобразование графиков

a) Вертикальный сдвиг б) Горизонтальный сдвиг в) Отражение г) Вертикальное растяжение и сжатие г) Горизонтальное растяжение и сжатие

Графики элементарных функций

Экспоненциальное, радикальное, рациональное, абсолютное значение

.

.

ГЛАВА 6 ПОЛИНОМИЛЫ

1) АЛГЕБРАИЧЕСКИЕ ВЫРАЖЕНИЯ

2) ПОЛИНОМЫ

-Равные полиномы Vid 1 ,

9

9

9 полинома

9 ,

-Полиномиальные функции Vid 1 ,

A. ДОБАВЛЕНИЕ ПОЛИНОМИЛОВ

B.ВЫЧИТАНИЕ ПОЛИНОМИЛОВ

C. УМНОЖЕНИЕ ПОЛИНОМОВ Vid 1 ,

D. РАЗДЕЛЕНИЕ ПОЛИНОМИЛОВ

Разделение двух многочленов (деление многочленов в длину) Vid 1 ,

3. Синтетическое деление (метод Хорнера)

a. Делим на x — c

b.Деление на ax + b

4. Нахождение остатка Vid 1 ,

5. Нахождение остатка от деления по x n + a

E. БИНОМИАЛЬНОЕ РАСШИРЕНИЕ ПОЛИНОМОВ Vid 1 ,

3) БИНОМИАЛЬНОЕ РАСШИРЕНИЕ

4) ОСНОВНЫЕ ИДЕНТИФИКАЦИИ

-Sial бином

-различие двух квадратов (факторизация)

-сумма двух кубов (факторизация)

-различие двух кубов (факторизация) (смешанный)

5) ФАКТОРИЗАЦИЯ ПОЛИНОМИЛОВ

-вычисление a Общий множитель

-Факторинг по группировке

-Факторизация идентификаторов (Вы можете найти их на стр. предыдущая часть.Последние три основных тождества относятся к факторизации.)

-Факторинг триномов Видео 12 , Видео 13 , с использованием метода полного квадрата

* Все методы факторизации смешаны Vid 1 ,

6) УПРОЩЕНИЕ РАЦИОНАЛЬНЫХ ВЫРАЖЕНИЙ С ПОЛИНОМИЛАМИ Vid 1 , полезные видеоролики видео 26,

-Сложение и вычитание рациональных выражений Vid 1 ,

ГЛАВА 7 9281 ) РЕШЕНИЕ КВАДРАТИЧЕСКИХ УРАВНЕНИЙ

a) Факторинг

b) Завершение Perfect Square

c) Квадратичная формула

d) Дискриминант

-все смешанные методы ,

e) Решение P проблемы

2) ТЕОРЕМА ВИЕТА Vid 1 ,

3) ВЫВОД КВАДРАТИЧЕСКИХ УРАВНЕНИЙ

4) УРАВНЕНИЯ, СВОДИМЫЕ К КВАДРАТИЧЕСКИМ УРАВНЕНИЯМ (с изменением формы 9011

9

) Неизвестно) (с заменой на t) Vid 1 ,

b) Уравнения, включающие продукты и коэффициенты Vid 1 ,

c) Уравнения, включающие радикалы

d) Уравнения, включающие Абсолютное значение

Системы квадратичных уравнений

5) КВАДРАТИЧЕСКИЕ НЕРАВЕНСТВА Vid 1 ,

Неравенства, сводимые к квадратичным 000 Vid 1 , СИСТЕМЫ НЕРАВЕНСТВА

7) ГРАФИКИ КВАДРАТИЧЕСКИХ ФУНКЦИЙ

— Построение графика Vid 1 ,

ГЛАВА 8 КОМПЛЕКСНЫЕ НОМЕРА

1) A

1)

)

2)

2) B

A)

1)

2)

B)

СЧЕТ ГЛАВНОЙ ПРОПИТКИ

1) Принципы подсчета

2) Перестановка

3) Комбинация

4) Вероятность

ГЛАВА 10 ПЕРИМЕТРЫ И ОБЛАСТИ ПЛОСКИХ ФИГУР

8 1 СВОЙСТВА ПЛОЩАДКИ

2) УГЛЫ ПОЛИГОНА

3) КВАДРИЛА ТЕРРАЛЫ И ВИДЫ ЧЕТЫРЬКИХ МОДУЛЕЙ

A.ПАРАЛЛЕЛОГРАММА

B. ПРЯМОУГОЛЬНИК

C. КВАДРАТ

D. RHOMBUS

E. KITE

F. TRAPEZOID

9028

ПОНЯТИЯ СТАТИСТИКИ

2) ПРЕДСТАВЛЕНИЕ И ИНТЕРПРЕТАЦИЯ ДАННЫХ

3) СТАТИСТИЧЕСКИЕ МЕРЫ

1. Среднее значение

2.Медиана

3. Режим

4. Диапазон

С самого начала я наблюдал только несколько. Эти видео объясняют все, даже то, что вам не нужно в старшей школе. Вы можете посмотреть их, чтобы получить общее представление. Видео 0, Видео 1, Видео 2, Видео 3, Видео 4, Видео 5, Видео 6, Видео 7, Видео 8, Видео 9, Видео 10, Видео 11, Видео 12, Видео 13, Видео 14, Видео 15, Видео 16, Видео 17,

1) Распределение частот Видео 1, Графическая гистограмма, Точечное видео 2, Групповое видео частоты 3, Видео 4, Среднее — Медианное — Режим видео 5, Видео 6, Видео 7,

2)? Процентили и дисперсия Видео 8, Видео 9, Видео 10, у нас нет этой части в нашей книге

3) Диапазоны и стандартное отклонение Видео 11, Видео нормального распределения 12, Видео 13,

.

.

.

ГЛАВА 12 ЛОГИКА

Видео 1,

1) ПРЕДЛОЖЕНИЯ

Истинные значения предложений

Эквивалентные предложения

Отрицание предложений

Таблицы истинных утверждений СОСТАВНЫЕ ПРЕДЛОЖЕНИЯ

a) Конъюнкция

b) Дизъюнкция

Общие свойства конъюнкции и дизъюнкции:

c) Условные

d) Двузначные

-Результаты поиска составных предложений с номерами 1 ,

— Упрощение сложных предложений буквами Vid 1 , Vid 2 ,

3) ТАУТОЛОГИЯ И ПРОТИВОРЕЧИВОСТЬ

— Противоречие тавтологии с таблицей истинности Vid 1 ,

4) ЭЛЕКТРИЧЕСКИЕ ЦЕПИ (необязательно)

5) МАТЕМАТИЧЕСКАЯ ИНДУКЦИЯ (необязательно)

.

Квадратичная формула — Повторно опубликована в Википедии // WIKI 2

формула, которая дает решения квадратного уравнения

Квадратичная функция с корнями x = 1 и x = 4.

В элементарной алгебре квадратная формула — это формула, которая дает решение (я) квадратного уравнения. Существуют и другие способы решения квадратного уравнения вместо использования квадратной формулы, такие как факторинг (прямое разложение, группировка, метод AC), завершение квадрата, построение графиков и другие.{2} -4ac}}} {2a}}}

Каждое из этих двух решений также называется корнем (или нулем) квадратного уравнения. Геометрически эти корни представляют собой значения x , при которых любая парабола , явно заданная как y = ax 2 + bx + c , пересекает ось x . [3]

Помимо того, что это формула, которая дает нули любой параболы, квадратная формула также может использоваться для определения оси симметрии параболы, [4] и количества действительных нулей, квадратное уравнение содержит. [5]

Энциклопедия YouTube

  • 1/5

    Просмотры:

    1682945

    234 668

    2307721

    135 318

    42 319

  • Как пользоваться формулой корней квадратного уравнения | Полиномиальные и рациональные функции | Алгебра II | Хан Академия

  • Как решить квадратные уравнения с помощью квадратной формулы

  • Решите квадратные уравнения, используя квадратную формулу

  • Как использовать квадратичную формулу для решения уравнений

  • Использование формулы корней квадратного уравнения для решения уравнения

Содержание

Эквивалентные составы

Квадратичная формула также может быть записана как

x = −b2a ± b2−4ac4a2, {\ displaystyle x = {\ frac {-b} {2a}} \ pm {\ sqrt {\ frac {b ^ {2} -4ac} {4a ^ {2} }}} \ \,}

, который можно упростить до

x = −b2a ± (b2a) 2 − ca. {2 } — {\ frac {c} {a}} {\ Big |}}} \ \.{2} + 2b_ {2} x + c = 0}, где b2 = b / 2 {\ displaystyle b_ {2} = b / 2}. [7]

Эти альтернативные параметризации приводят к несколько другим формам решения, которые в остальном эквивалентны стандартной параметризации.

Вывод формулы

В литературе доступно множество различных методов вывода формулы квадратного уравнения. Стандартный — это простое выполнение техники завершения квадрата. [8] [9] [10] [11] Альтернативные методы иногда проще, чем заполнение квадрата, и могут предложить интересное понимание других областей математики.{2} -4ac \}}} {2a}} \ \.}

Есть много альтернатив этого вывода с небольшими отличиями, в основном касающимися манипуляции с {\ displaystyle a}.

Метод 2

В большинстве текстов по алгебре, опубликованных за последние несколько десятилетий, учат завершать квадрат, используя последовательность, представленную ранее:

  1. Разделите каждую сторону на {\ displaystyle a}, чтобы получился полином однозначный. {2}} к обеим сторонам, чтобы завершить квадрат.{2} -4ac}}} {2a}} \. \ End {align}}}

    Этот вывод квадратной формулы является древним и был известен в Индии, по крайней мере, еще в 1025 году. [13] По сравнению с выводом в стандартном использовании, этот альтернативный вывод позволяет избежать дробей и квадратов дробей до последнего шага и, следовательно, не требует перегруппировки после шага 3 для получения общего знаменателя в правой части. [12]

    Метод 3

    Аналогично методу 1, разделите каждую сторону на {\ displaystyle a} , чтобы получился полином с левой стороны монич. (Т. Е.{2}} {4}} — C}}}

    Путем повторного выражения B {\ displaystyle B} и C {\ displaystyle C} обратно в ba {\ displaystyle \ textstyle {\ frac { b} {a}}} и ca {\ displaystyle \ textstyle {\ frac {c} {a}}} соответственно, тогда можно получить формулу корней квадратного уравнения. [ необходима ссылка ]

    Путем замены

    Другой метод — решение заменой. [14] В этом методе мы подставляем x = y + m {\ displaystyle x = y + m} в квадратичную, чтобы получить:

    а (y + m) 2 + b (y + m) + c = 0.{2} + bm + c \ right) = 0 \ \.}

    Мы еще не наложили второе условие на y {\ displaystyle y} и m {\ displaystyle m} , поэтому теперь выбираем m {\ displaystyle m} , так что средний член исчезает. То есть 2 am+b=0 {\ displaystyle 2 am+b=0} или m = −b2a {\ displaystyle \ textstyle m = {\ frac {-b} {2a}}} . Вычитая постоянный член из обеих частей уравнения (чтобы переместить его в правую часть) и затем разделив на {\ displaystyle a} , получаем:

    y2 = — (am2 + bm + c) а.{2} -4ac}}} {2a}} \ \.}

    По резольвентам Лагранжа

    Альтернативный способ получения квадратичной формулы — это метод резольвент Лагранжа, [16] , который является ранней частью теории Галуа. [17] Этот метод можно обобщить, чтобы получить корни кубических многочленов и многочленов четвертой степени, и приводит к теории Галуа, которая позволяет понять решение алгебраических уравнений любой степени в терминах группы симметрии их корней, группы Галуа.{2} — (\ alpha + \ beta) x + \ alpha \ beta \ \,}

    , где p = — ( α + β ) и q = αβ .

    Поскольку порядок умножения не имеет значения, можно поменять местами α и β , и значения p и q не изменятся: можно сказать, что p и q являются симметричными многочленами. в α и β . Фактически, они являются элементарными симметричными многочленами — любой симметричный многочлен от α и β может быть выражен через α + β и αβ Подход теории Галуа к анализу и решению многочленов: коэффициенты полинома, которые являются симметричными функциями от корней, можно ли «нарушить симметрию» и восстановить корни? Таким образом, решение полинома степени n связано со способами перестановки («перестановки») n членов, которая называется симметричной группой из n букв и обозначается S n .Для квадратичного многочлена единственный способ переставить два члена — это поменять их местами («транспонировать»), и, таким образом, решение квадратного многочлена просто.

    Чтобы найти корни α и β , рассмотрим их сумму и разность:

    r1 = α + βr2 = α − β. {\ Displaystyle {\ begin {align} r_ {1} & = \ alpha + \ beta \\ r_ {2} & = \ alpha — \ beta \ \. \ End {выровнено}}}

    Они называются резольвентами Лагранжа полинома; обратите внимание, что один из них зависит от порядка корней, что является ключевым моментом.Можно восстановить корни из резольвент, обратив приведенные выше уравнения:

    α = 12 (r1 + r2) β = 12 (r1 − r2). {\ Displaystyle {\ begin {align} \ alpha & = \ textstyle {\ frac {1} {2}} \ left (r_ {1 } + r_ {2} \ right) \\\ beta & = \ textstyle {\ frac {1} {2}} \ left (r_ {1} -r_ {2} \ right) \ \. \ end {выравнивается} }}

    Таким образом, решение для резольвент дает исходные корни.

    Теперь r 1 = α + β является симметричной функцией в α и β , поэтому ее можно выразить через p и q , а на самом деле r 1 = — p , как указано выше.Но r 2 = α β не является симметричным, поскольку переключение α и β дает — r 2 = β α (формально это называется групповое действие симметрической группы корней). Поскольку r 2 не является симметричным, его нельзя выразить через коэффициенты p и q , поскольку они симметричны по корням и, следовательно, любое полиномиальное выражение, включающее их.{2} -4q}} \ right)}

    , которая является формулой корней квадратного уравнения. Подстановка p = b / a , q = c / a дает обычную форму для случаев, когда квадратичная величина не является монической. Резольвенты можно распознать как r 1 /2 = — p /2 = — b /2 a является вершиной, а r 2 2 = p 2 — 4 q — дискриминант (монического полинома).

    Аналогичный, но более сложный метод работает для кубических уравнений, в которых есть три резольвенты и квадратное уравнение («разрешающий многочлен»), связывающее r 2 и r 3 , которое можно решить с помощью квадратное уравнение, и аналогично для уравнения четвертой степени (степень 4), разрешающий многочлен которого является кубикой, которая, в свою очередь, может быть решена. [16] Тот же метод для уравнения пятой степени дает полином степени 24, что не упрощает задачу, и, фактически, решения уравнений пятой степени в целом не могут быть выражены с использованием только корней.

    Историческая застройка

    Самыми ранними методами решения квадратных уравнений были геометрические. Вавилонские клинописи содержат задачи, сводимые к решению квадратных уравнений. [18] Египетский берлинский папирус, восходящий к Среднему царству (2050 г. до н.э. — 1650 г. до н.э.), содержит решение двухчленного квадратного уравнения. [19]

    Греческий математик Евклид (около 300 г. до н.э.) использовал геометрические методы для решения квадратных уравнений в Книге 2 своего влиятельного математического трактата Elements . [20] Правила квадратных уравнений появляются в китайской книге Девять глав по математике около 200 г. до н. Э. [21] [22] В своей работе « Arithmetica » греческий математик Диофант (около 250 г. н.э.) решил квадратные уравнения методом более узнаваемым алгебраическим, чем геометрическая алгебра Евклида. [20] Его решение дает только один корень, даже если оба корня положительны. [23]

    Индийский математик Брахмагупта (597–668 н.э.) подробно описал квадратичную формулу в своем трактате Brāhmasphuṭasiddhānta , опубликованном в 628 году нашей эры, [24] , но написанном словами, а не символами. [25] Его решение квадратного уравнения ax 2 + bx = c выглядело следующим образом: «К абсолютному числу, умноженному на четырехкратный [коэффициент] квадрата, добавьте квадрат [коэффициент] среднего члена; квадратный корень из того же, за вычетом [коэффициента] среднего члена, деленный на удвоенный [коэффициент] квадрата, является значением ». [26] Это эквивалентно:

    х = 4ac + b2 − b2a.{2}}} — b} {2a}} \ \.}

    Персидский математик IX века Мухаммад ибн Муса аль-Хваризми решал квадратные уравнения алгебраически. [27] Квадратичная формула, охватывающая все случаи, была впервые получена Саймоном Стевином в 1594 году. [28] В 1637 году Рене Декарт опубликовал La Géométrie , содержащий частные случаи квадратичной формулы в той форме, которую мы знаем сегодня. [29]

    Значимые применения

    Геометрическое значение

    График y = ax 2 + bx + c , где a и дискриминант b 2 — 4 ac положительны, с корнями
    • и y -пересечение красным
    • Вершина и ось симметрии синим
    • Фокус и направляющая розового цвета

    В терминах координатной геометрии парабола — это кривая, координаты которой ( x , y ) описываются полиномом второй степени, т.е.{2} + a_ {1} x + a_ {0} \ \,}

    , где p представляет полином степени 2, а a 0 , a 1 и a 2 ≠ 0 — постоянные коэффициенты, нижние индексы которых соответствуют степени их соответствующих членов. {2} -4ac \}} {2a}}}

    ось симметрии отображается как линия x = — b /2 a .Другой член, √ b 2 — 4 ac /2 a , дает расстояние, на котором нули находятся от оси симметрии, где знак плюс представляет расстояние вправо, а знак минус представляет расстояние слева.

    Если бы этот член расстояния уменьшился до нуля, значение оси симметрии было бы значением x единственного нуля, то есть существует только одно возможное решение квадратного уравнения. Алгебраически это означает, что √ b 2 — 4 ac = 0 или просто b 2 — 4 ac = 0 (где левая часть обозначается как дискриминант ).Это один из трех случаев, когда дискриминант указывает, сколько нулей будет у параболы. Если дискриминант положительный, расстояние будет отличным от нуля, и будет два решения. Однако существует также случай, когда дискриминант меньше нуля, и это указывает на то, что расстояние будет мнимым — или некоторым кратным комплексной единицы i , где i = √ − 1 — и нули параболы будут комплексными числами. Комплексные корни будут комплексно сопряженными, где действительная часть комплексных корней будет значением оси симметрии.Реальных значений x там, где парабола пересекает ось x , не будет.

    Размерный анализ

    Если константы a , b и / или c не являются безразмерными, то единицы x должны быть равны единицам b / a , из-за требования, что ax 2 и bx согласовывают свои единицы измерения. Кроме того, по той же логике единицы c должны быть равны единицам b 2 / a , что можно проверить без решения для x . Рене Декарт. Геометрия .

Эта страница последний раз была отредактирована 9 июля 2021 в 00:41

) Почему в квадратных уравнениях обычно два решения

  • Почему в квадратных уравнениях всегда есть как минимум два решения

    Почему в квадратных уравнениях всегда есть как минимум два решения?

  • Как узнать, будет ли у квадратного уравнения одно, два или ни одного решения?

    Как узнать, будет ли у квадратного уравнения одно, два или ни одного решения? Как найти квадратное уравнение, если вам дано только решение? Возможно ли иметь разные квадратные уравнения с одним и тем же решением? Объяснять.Предложите одноклассникам одно или два решения, с помощью которых они должны составить квадратное уравнение.

  • как узнать, будет ли у квадратного уравнения одно, два или ни одного решения

    как узнать, будет ли у квадратного уравнения одно, два или ни одного решения? Если бы только было дано решение, как найти квадратное уравнение? Возможно ли иметь разные квадратные уравнения с одним и тем же решением?

  • Предположим, что x = a и x = e — два решения квадратных уравнений x2…

    Предположим, что x = a и x = e — два решения квадратных уравнений x2 + px = q. Используйте метод Вите, чтобы найти взаимосвязь между a, e. Предположим, что x = a, e — два решения кубического уравнения x3 + px = q. Используйте подход Вите, чтобы найти связь между a, e. Примечание. Если вы его не помните, вы можете найти факторизацию aº + b3 и a3 23 –…

  • DQuestion 19 28 pts Задача 2. Квадратичные уравнения Квадратичное уравнение имеет вид: a bc0 Два решения задаются формулой: 2a Напишите программу с циклом, который а) решает квадратное уравнение…

    DQuestion 19 28 pts Задача 2. Квадратичные уравнения Квадратичное уравнение имеет вид: a bc0 Два решения задаются формулой: 2a Напишите программу с циклом, который а) решает квадратные уравнения с коэффициентами, считываемыми с терминала, б) визуализирует соответствующую квадратичную функцию az2 brc0 с помощью модуля matplotlib. Инструкции Запишите весь код для этой проблемы в файл p2.py 1. Программа состоит из основного цикла whtle (бесконечного цикла), в котором пользователь …

  • Выберите любые два целых числа от -12 до +12, которые станут решениями системы двух уравнений.

    1.Выберите любые два целых числа от -12 до +12, которые станут решениями системы двух уравнений. 2. Напишите два уравнения, решениями которых являются два ваших целых числа. Покажите, как вы строили уравнения, используя целые числа. Ваше решение и уравнения не должны совпадать с решениями других студентов или учебником. Возможности бесконечны. 3. Решите вашу систему уравнений методом сложения / вычитания. Убедитесь, что вы показываете необходимые 5 шагов. Использовать…

  • Подробно объясните, как определить значение независимой переменной в квадратичной зависимости, если значение зависимой переменной известно.

    1.Объясните подробно, как определить значение независимой переменной в квадратичной зависимости, если значение зависимой переменной известно. Пожалуйста, проверьте мой ответ — вы замените значение зависимой переменной, а затем вы решите независимую переменную. (это правильно?) 2. Какое наибольшее количество решений может иметь квандратное уравнение? Объясните на примере, почему все решения уравнений не могут быть разумными ответами на …

  • 2 примера пожалуйста Некоторые задачи со словами в этой главе включают одно «простое уравнение», обычно находящееся с помощью…

    2 примера пожалуйста Некоторые словесные задачи в этой главе включают одно «простое уравнение», обычно находящееся путем перевода простого соотношения между x и y (например, одно является двойным или что они прибавляют к заданному числу). Во втором линейном уравнении часто используется формула «скорость умножения на количество». Это верно для • задач о расстоянии (скорость.время = расстояние) • проблем со смесью (процентная концентрация раствора — количество концентрата) денежных проблем (стоимость одного элемента. Как много предметов…

  • Задача 4. Рассмотрим f (x) = x5 + x4 + 2×3 + 3×2 + 4x + 5 ∈ Q [x], и наша цель — определить, соответствует ли f i …

    Задача 4. Рассмотрим f (x) = x5 + x4 + 2×3 + 3×2 + 4x + 5 ∈ Q [x] и наша цель — определить, неприводима ли f над Q. Мы вычисляем f (1), f (−1), f (5), f (−5) и убедитесь, что ни один из них не равен нулю. По Rational Теорема о корнях, f не имеет корня в Q. Итак, если f приводима над Q, она нельзя разложить на произведение линейного многочлена и полином четвертой степени (т.е. многочлен от …

  • Некоторые задачи со словами в этой главе включают одно «простое уравнение», обычно находящееся путем перевода простого …

    Некоторые словесные задачи в этой главе включают одно «простое уравнение», обычно находящееся путем перевода простого соотношения между x и y (например, одно является двойным или что они прибавляют к заданному числу). Во втором линейном уравнении часто используется формула «скорость умножения на количество». Это верно для • задач с расстоянием (скорость времени = расстояние) • проблем со смесью (процентное содержание раствора = количество концентрата) • денежных проблем (стоимость одного элемента = сколько элементов…

  • Международная математическая олимпиада

    Международная математическая олимпиада Другие научные олимпиады: Физика, Химия, Информатика, Биология, Астрономия, Лингвистика
    Этот сайт официально НЕ связан с IMO
    Эта страница больше не поддерживается.
    Считайте это пещерой для археологических экскурсий.
    Используйте информацию по своему усмотрению.
    Ссылка на официальный сайт ИМО

    Международная математическая олимпиада (ИМО, также известная как Международная математическая олимпиада) ежегодная олимпиада по математике для старшеклассников [Статья ИМО в Википедии].Это один из старейших Международные научные олимпиады. Первая IMO была проведена в Румынии в 1959 году. Задачи приходят из разных областей математики, такие, которые включены в учебные программы по математике в средних школах. Находя решения этих проблем, однако требует исключительных математических способности и отличные математические знания участников.

    Темы охватывали (см. Ненаписанную программу и учебные пособия на веб-сайте математической олимпиады Аркадия Слинко, в настоящее время переносится):

    • Теория чисел, в том числе
      • Основные теоремы арифметики
      • Линейные и квадратные диофантовы уравнения, включая уравнение Пелла
      • Арифметика вычетов по модулю n , теоремы Ферма и Эйлера
    • Алгебра, в том числе
      • Основные теоремы по алгебре, e.грамм. неравенства, факторизация многочлен в произведение неприводимых многочленов
      • Симметричные многочлены многих переменных, теорема Виета
    • Комбинаторика, в том числе
    • Геометрия, включая
      • Свойства ортоцентра, линия Эйлера, девятиконечная окружность, Линия Симсона, неравенство Птолемея, Сева и Менелай и т. Д.
    Исключенные темы:
    • Исчисление (!)
    • Комплексные числа (хотя и присутствовали в прошлом)
    • Инверсия в геометрии
    • Твердая геометрия (хотя присутствовала в прошлом, может вернуться)

    Обычный размер официальной делегации в ИМО составляет (максимум) шесть студентов-конкурентов и (максимум) два лидера.Официальной « команды » нет. Студенты-соискатели пишут две работы в последовательные дни, каждая статья состоит из трех вопросов. Каждый вопрос стоит семь баллов. (Предыдущая информация взята из Обзор IMO предоставлено страной, принимающей IMO’95, Канадой; также см. ниже.) Возможна общая оценка 42 балла. Награды определяются следующим образом:

    • ЗОЛОТАЯ МЕДАЛЬ: 1/12 набравших наибольшее количество баллов получают золотые медали
    • СЕРЕБРЯНАЯ МЕДАЛЬ: следующие 2/12 очков получают серебряные медали
    • БРОНЗОВАЯ МЕДАЛЬ: следующие 3/12 очков получают бронзовые медали
    • ПОЧЕТНОЕ НАПОМИНАНИЕ: любой участник, получивший наивысший балл 7 по любому вопросу, но не получивший медаль, награждается почетное упоминание

    В моем отчете по IMO за 2002 год представлены дополнительные сведения о том, как работает IMO.

    Список рекомендованной литературы по математике для самообучение медленно развивается.


    Содержание

    Без сомнения, эти списки веб-адресов далеко не полные. если ты отправить соответствующие веб-адреса не перечисленные здесь, то они будут включены.


    Официальный веб-сайт ИМО <<<<
    Новый сайт об ИМО, включая архив проблем и результатов.
    Логотипы
    Обзор всех доступных логотипов

    Положения
    Положения, относящиеся к ИМО
    Настоящие Правила определяют ИМО с?.Новые аспекты — это Консультативный совет ИМО и Целевой фонд ИМО.
    Общий (но не официальный) сайт IMO

    Будущие ИМО
    IMO 2009
    Пятидесятая ИМО будет проходить в Германии в Бремен 10-22 июля 2009 г.
    IMO 2010
    51-я ИМО будет проходить в Казахстане в ? на ?-? ?, 2010.
    IMO 2011
    52-я ИМО будет проходить в Нидерландах в ? на ?-? ?, 2011.

    Прошлые ИМО
    IMO 2008 [локальные файлы]
    49-я ИМО была проведена в Испании в Мадрид 10-22 июля 2008 г.(день заезда конкурсантов 14 июля).
    IMO 2007 [локальные файлы: логотип | проблемы день 1, день 2 ]
    48-я ИМО была проведена во Вьетнаме в Ханой, 19-31 июля 2007 г. (день заезда конкурсантов — 23 июля).
    IMO 2006 [локальные файлы: о | логотип | проблемы день 1, день 2 | решения | полученные результаты ]
    47-я ИМО была проведена в Словении в Любляна, 6-18 июля 2006 г.
    IMO 2005 [локальные файлы: | о | логотип | правила | страны | проблемы день 1, день 2 | полученные результаты ]
    46-я ИМО была организована Мексикой в Мерида, Юкатан, 8-19 июля 2005 г.
    IMO 2004 [логотип | правила | проблемы день 1, день 2 | все данные | Отчет по Великобритании, вкл. проблемы ]
    45-я ИМО была проведена в Греции в Афины, 6-18 июля 2004 г.
    IMO 2003 [старый адрес | английский | логотип | результаты (Excel) | проблемы 1 день, день 2 | решения ]
    44-я ИМО была проведена в Японии в Токио, 7-19 июля 2003 г. Крайний срок подачи проблем — 15 февраля 2003 г.
    IMO 2002 [логотип | проблемы в PDF (Нидерландский язык) | полученные результаты | статистика | личный отчет | шорт-лист является конфиденциальным до IMO2003 ]
    43-я ИМО была проведена в Соединенном Королевстве в Глазго, 19-30 июля 2002 г.
    IMO 2001, Сайт оценки [логотип | проблемы в PDF | проблемы и решения как Mathematica Notebook | полученные результаты | список рассылки | список | книга ]
    42-я ИМО была проведена в Соединенных Штатах Америки в Вашингтон, округ Колумбия, 1-14 июля 2001 г.
    IMO 2000 [логотип | нормативно-правовые акты | проблемы день 1 | проблемы день 2 | проблемы в PDF | полученные результаты | медали | список рассылки ]
    41-я ИМО была проведена в Южной Корее в Тэджоне. 13-25 июля 2000 г.
    IMO 1999 [логотип | проблемы день 1 | проблемы день 2 | полученные результаты | список рассылки ]
    С 10 по 22 июля 1999 г. в Румынии в Бухаресте проходила 40-я конференция IMO.
    IMO 1998 [логотип | проблемы | полученные результаты ]
    39-я ИМО была организована на Тайване в Тайбэе 10-21 июля 1998 года.
    IMO 1997 [логотип | проблемы день 1 | проблемы день 2 | полученные результаты ]
    38-я ИМО был организован Аргентиной в Мар-дель-Плата 18-31 июля 1997 года.Есть список рассылки IMO97.
    IMO 1996 [логотип | проблемы ]
    37-я ИМО была проведена в Индии в Мумбаи (Бомбей) 5-17 июля 1996 г.
    Контактная информация:
    Профессор А.М. Vaidya
    IMO-Cell, Школа математики,
    Tata Institute of Fundamental Research
    Homi Bhabha Road, Mumbai (Bombay) -400005
    India

    Телефон: 91 (022) 2152971, 2152311, 2188654
    Телефон: 91 (022) 2152110/2152181
    Электронная почта: [email protected] или imo @ math.tifr.res.in

    IMO 1995 [логотип | нормативно-правовые акты | проблемы ]
    36-я ИМО была организована в Канаде в Торонто 10-28 июля 1995 года.
    IMO 1994 [логотип | проблемы и результаты]
    Тридцать пятая IMO проходила в Гонконге 12-19 июля 1994 г.
    IMO 1993 [логотип | проблемы день 1 | проблемы день 2 ]
    34-я ИМО была организована в Турции в Стамбуле 12-19 июля 1993 года.

    Журналы и другая информация
    IMOnet
    Сеть для участников IMO.Имеет информацию о трех списках рассылки (для проблем IMO’99, IMO2000 и IMO).
    Новости IMO в MAA
    Математическая ассоциация Америки (MAA) содержит некоторую информацию об ИМО и публикует книги с ИМО проблемы.
    Результаты IMO
    На этом сайте есть некоторые статистические данные об оценках на IMO в 1993 году (кроме 1994). Поддерживается Джозефом Майерсом.
    KöMaL — Математические и физические Журнал для средних школ
    Более ста лет назад, Днепр Араны, учитель средней школы из города Гыр, решил основать математический журнал для старшеклассников.Его целью было «дать множество примеров студентам и учителям». Первое издание журнала вышло 1 января 1894 года.
    MathPro Press
    MathPro Press специализируется на публикации сборников и указателей задач математики.
    WFNMC
    Всемирная федерация Национальные соревнования по математике (WFNMC) издает журнал под названием Соревнования по математике . Также см. Австралию ниже.
    M&IQ
    The Mathematics and Informatics Quarterly — международный журнал, который:
    • Публикует статьи, заметки, проблемы и решения в школе. математика и информатика.
    • Посвящается учителям и студентам, интересующимся клубы математики и информатики, олимпиады и соревнования.
    В мире математики
    Этот новый ежеквартальный журнал предназначен для школьников, студентов и учителей. интересуется математическими задачами и головоломками а также в истории математики и ее новых направлений.
    Бюллетень SIPROMA (на испанском языке)
    СИПРОМА — это « Sociedad Iberoamericana para la Promocin de la Matemtica ».
    Кангуру без границ / Кенгуру без границ: международный конкурс математиков (французский сайт)
    Кангуру без границ (KSF) является международная олимпиада по математике для школьников разного уровня. История и правила (старые)
    Математические олимпиады для начальной и средней школы
    Создано в 1977 году доктором Джорджем Ленчнером, всемирно известный педагог по математике, Математические олимпиады стали достоянием общественности в 1979 году. В 2000 году более 120000 студентов из 5000 команд по всему миру участвовали в олимпиадах, представляя 26 стран.
    Международная олимпиада по живой математике <<<<< N E W
    Живая математика (Южная Африка) организует Международную олимпиаду по живой математике для детей от 6 до 13 лет (K-7). Смотрите: События, Олимпиада.
    Чемпионат мира по ментальному расчету
    Азиатско-Тихоокеанская олимпиада по математике
    Задачи на английском и французском языках.
    Балканская математическая олимпиада
    Ссылки пока отсутствуют.
    Математическая олимпиада в районе залива
    Математическая область залива Олимпиада (БАМО) — это соревнование для старшеклассников, спонсируемое совместно Институт математических наук (ИИГС), Американский институт Математика (AIM), Калифорнийский университет в Беркли (UCB), и Университет Сан-Франциско (USF).
    Колорадская математическая олимпиада
    Математическая олимпиада в Колорадо (CMO) это крупнейший математический конкурс сочинений в США, ежегодно за призы соревнуются от 600 до 1000 участников.
    Иберо-американская олимпиада по математике
    Домашняя страница (на испанском языке) Olimpíadas Iberoamericanas de Matemática (OIM).
    Математическая лига (США и Канада)
    Математическая лига посвящена приносить учащимся сложные материалы по математике и специализируется в олимпиадах по математике (для средних школ США и Канады), книгах, и компьютерное программное обеспечение.
    Аргентина
    Олимпиада Математика Аргентина (OMA) — это Аргентинская олимпиада по математике. В настоящее время информация доступна только на испанском языке.
    Австралия
    Австралийский фонд математики (AMT) национальная некоммерческая организация, проводящая олимпиады по математике. в Австралии. Включает информацию о WFNMC.
    Бельгия
    Информация об олимпиадах по математике в Бельгии:
    Бразилия [также см. здесь]
    Информация о Бразильская олимпиада по математике (OBM = Olimpíada Brasileira de Matemática).Все материалы только на португальском языке.
    Болгария
    Информация о Национальная математическая олимпиада в Болгарии как часть Болгарские соревнования в Математика и информатика.
    Канада
    Информация о Канадской олимпиаде математиков (CMC) поддерживается Рут Малиновски ([email protected]).

    Канадское математическое общество также есть Домашняя страница Олимпиады.

    Колумбия
    Домашняя страница WWW колумбийских олимпиад (на испанском языке), по математике, физике и информатике, поддерживается Фернандо Вега Саламанка (fvega @ zulima.uanarino.edu.co).
    Эл. Почта: [email protected]
    Чешская Республика
    Дания
    Эстония
    Информация о Эстонская Матемаатика Олимпиадид (также на эстонском языке), поддерживается Уве Нуммерт ([email protected]). Включает информацию IMO (на английском языке).
    Франция
    Информация о олимпиадах по математике во Франции, включает французские переводы наборов задач ИМО и рекомендована французская математическая литература для подготовки ИМО.
    Германия
    Verein Mathematik-Olympiaden e.V. координирует немецкие математические олимпиады (DeMO). Также доступна информация о Немецкий Математическая олимпиада 1995.
    Ирландия
    Информация об участии Ирландии в Международном Математические олимпиады и (в конечном итоге) копии Олимпиада по ирландской математике вопросы и фотографии. Контактное лицо: Гордон Лесселлс ([email protected]).
    Израиль
    Италия
    Информация об итальянской (?) Математической олимпиаде (на итальянском языке).Контактная электронная почта: [email protected]
    Япония
    Информация об ИМО в целом (начиная с 34-го) и Фонд математической олимпиады Японии. Поддерживает Наомаса Маруяма ([email protected]).
    Люксембург
    Мексика
    Информация (на испанском языке) о мексиканской олимпиаде по математике (OMM) и наборы задач (в основном на испанском) различные (региональные) олимпиады по математике. Эл. Почта: [email protected].Поддержкой занимается Фелипе Леметр Карабиас. ([email protected]).

    Существует также неофициальная страница мексиканской математической олимпиады. (на испанском языке), поддерживается Педро Санчес.

    Пьер Ферма Математический конкурс (на испанском: Concurso de Matemáticas Pierre Ферма).

    Нидерланды
    Информация о Голландская олимпиада по математике (теперь также на английском языке), поддерживается Томом Верхоффом ([email protected]).
    Новая Зеландия
    Информация о новинке Олимпиады по математике в Зеландии
    Норвегия
    Нильс Хенрик Абельс Соревнование по математике отбирает студентов для участия в ИМО.Этот сайт также содержит наборы задач норвежской математической олимпиады. и ИМО, включая некоторые решения и результаты. Поддерживается Эйнар Андреас Рёдланд ([email protected]).
    Панама
    Перу
    Информация об олимпиаде по математике в Перу и олимпиадах Коно Сур, поддерживается Энрике Валериано ([email protected]).
    Польша
    Информация (включая наборы задач) по математическим олимпиадам в которых участвуют польские старшеклассники (в частности: Польская математическая олимпиада, австрийско-польская математическая олимпиада и Балтийское командное соревнование).Поддерживается Кшиштоф Хелмински ([email protected]) а также Вальдемар Помпе ([email protected]).
    Португалия
    Olimpíadas Portuguesas de Matemática.
    Сингапур
    Информация об участии ИМО и Сингапура.
    Словакия
    Математическая олимпиада в Словакии.
    Словения
    Математические соревнования в Словении курирует DMFA.
    Южная Африка
    Южноафриканская математическая олимпиада.
    Испания
    Информация и наборы задач (на испанском языке) для математических олимпиад в которых участвуют испанские старшеклассники, в том числе олимпиада по испанской математике (Olimpiada Matemática Española = OME). Поддержкой занимается Кристобаль Снчес Рубио. ([email protected]).

    Неофициальная олимпиада по испанской математике страница поддерживается Рамон Эстебан Ромеро ([email protected]).

    Швейцария
    Информация об участии Швейцарии в IMO на английском, французском, немецком, итальянском языках.
    Соединенное Королевство
    Информация о Британская математическая олимпиада (BMO). Регистр ИМО Великобритании предлагает много исторической информации об участии Великобритании в ИМО. Просмотрите Отчеты лидеров.
    Связано: United Kingdom Mathematics Trust (УКМТ), зарегистрированная благотворительная организация который проводит в школах ряд дополнительных математических мероприятий.
    Уругвай
    США
    Информация о Математическая олимпиада США и ИМО.
    США: AMC
    Американские математические соревнования (AMC).
    США: USAMTS
    Поиск талантов в математике в США (USAMTS) — бесплатный Соревнования по математике открыты для всех учащихся средних и старших классов средней школы США. Это отличный конкурс для студентов, стремящихся решать олимпиадные задачи, поскольку многие проблемы требуют решения, основанного на доказательствах.
    Коллекция IMO ===== НОВИНКА =====
    Здесь вы найдете большую коллекцию олимпиадных задач из всех по всему миру.Вы также найдете всю информацию о Сборник ИМО , наиболее полный сборник задач, предложенных Международные математические олимпиады.
    Искусство решения проблем
    Искусство решения проблем содержит множество ресурсов для заядлых студентов-математиков в средняя и старшая школа. Есть много бесплатных ресурсов (помимо Forum), такие как статьи, учебник по LaTeX и онлайн-сессии Math Jam. Решатели олимпиадных задач со всего мира участвуют в решении задач обсуждения.На сайте также продается набор учебников по решению задач. и есть онлайн-школа.

    Включает проблемы и решения IMO с авторами.

    Советы профессора Шапиро по решению математических задач
    На этом веб-сайте описаны некоторые общие методы решения математических задач. Он намерен превратиться в интерактивный веб-сайт. Поддерживает Гарольд Шапиро.
    MathLinks.ro
    Сборники математических задач, частые (сложные!) Математические соревнования и большое сообщество по решению проблем с множеством онлайн-форм для обсуждения.Поддерживает Валентин Ворнику.
    Большая коллекция материалов для соревнований по математике
    Включает задачи по теории элементарных чисел (PEN), Темы в неравенствах (TIN) и Интернет-каталог для решения проблем (PSD). Архив поддерживает Ходжу Ли (он же «идея») в Республике Корея.
    PEN: Проблемы элементарной теории чисел
    Форум, посвященный сборнику задач теории чисел Ходжу Ли (см. выше).
    Сборник математических олимпиадных задач
    Архив поддерживает Ганс Вернаев (Ганс[email protected]) в Бельгии.
    Сайт математической олимпиады Аркадия Слинко (постепенно переносится)
    Содержит: сборник задач, учебные пособия, обучающие материалы, статьи, ежемесячная интернет-олимпиада по математике и информация о Новой Зеландии Математические олимпиады.
    Архив соревнований по математике
    Киран Кедлая (Массачусетский технологический институт, США, [email protected]) имеет Математика Архив соревнований с некоторыми из недавних (с 1991 года) Наборы задач ИМО в различных форматах, и информация о математической олимпиаде США (USAMO).Теперь у него есть отдельный Справочник математических задач.
    Соревнования по математике в Интернете
    Архив и ссылки поддерживаются Наоки Сато ([email protected]) в Торонто, Канада.
    OMAP: Онлайн-математические задачи
    База данных проблем, возникших на различных национальных и международные математические соревнования по всему миру, включая подробную информацию о Математическом конкурсе Патнэма. Поддержкой занимается [email protected].
    Советский Союз 1961-1986
    задачи Всесоюзных всесоюзных математических олимпиад (заключительная часть) 1961-1986 гг. переведены на английский язык и размещены в сети. Владимир Перцель (voldemar @ sagantec.co.il). Существует также простая версия ASCII и сжатый архив со всем: изображениями HTML, ASCII и GIF. [The оригинал страница и оригинал архив все еще доступен.]
    Олимпиада по математике Безумие
    Сборник математических головоломок, специально для тех, кто устали от не очень сложных проблем. Поддерживается [email protected].
    Конспект лекций по математике
    Грег Гэмбл ([email protected]).
    20000 морских проблем: Математические сокровища в Интернете
    Онлайн-справочник по математическим задачам.
    Домашняя страница Kalva
    Сборники задач, поддерживаемые Джоном Скоулзом.
    Математическая олимпиада Ле Тай Хоанга
    Страница математической олимпиады Ле Тай Хоанга содержит множество наборов математических задач, включая короткие списки для ИМО с 1983 по 1998 год.
    Математические головоломки Ника
    Сборник головоломок по геометрии, вероятности, теория чисел, алгебра, исчисление и логика. Подсказки предоставил вместе с ответами полностью проработанные решения и ссылки на связанные математические темы.Многие загадки элементарно в их заявлении, но все же сложно. Новые головоломки добавляются регулярно.
    Ключевой особенностью сайта является подробная экспозиция от первые принципы решения головоломки. Несколько из головоломки используются для демонстрации определенных математических концепции. См., Например, загадку 56, в которой тождество разбиения и головоломка 63, где теорема Птолемея позволяет найти удивительно простое решение. Дальнейшие ссылки предоставлены многие решения.
    AIMS [Мероприятия, объединяющие математику и естественные науки]
    AMS [Американское математическое общество]
    EMS [Европейское математическое общество]
    GAMS [Руководство по доступному математическому программному обеспечению]
    IMU [Международный математический союз]
    MAA [Математическая ассоциация Америки]
    Математические архивы
    PSTC [Центры задач по решению проблем]
    PSU [Penn State University]
    SIAM [Общество промышленной и прикладной математики]

    Разные ссылки


    Секретариат IOI / ioi-секретариат @ win.вт.нл

    Интернет-сервис предоставляется

    Математические заметки Джони: циклотомические многочлены

    В этом посте мы рассматриваем круговые многочлены, которые представляют собой особый класс многочленов с различными связями с теорией чисел, алгеброй и даже с возможностью построения правильных многоугольников с помощью линейки и циркуля. После вывода основных свойств круговых многочленов мы используем их, чтобы дать элементарное доказательство бесконечности простых чисел $ p \ Equiv 1 \ pmod n $, а также определить, какие правильные многоугольники можно построить.2-х + 1 $. Это уже дает намек на два интересных свойства круговых многочленов, а именно на то, что они имеют целые коэффициенты и, кроме того, неприводимы в $ \ mathbb {Z} [x] $, наборе многочленов с целыми коэффициентами (было бы заманчиво также предположить что коэффициенты всегда равны $ \ pm 1 $, но в целом это не так; $ n = 105 $ — пример наименьшего счетчика). Первое из этих утверждений сразу же.

    Предложение 2. Многочлены $ \ Phi_n (x) $ имеют целые коэффициенты.m-1} {\ prod ‘_ {d \ mid m} \ Phi_d (x)}, \ end {eqnarray} \]

    (где снова $ d = m $ исключено) многочлен $ \ Phi_m (x) $ является частным двух одночленных многочленов с целыми коэффициентами, скажем, $ f (x) $ и $ g (x) $. Когда мы выполняем длинное деление для $ f (x) $ и $ g (x), $ очевидно, что все шаги в длинном делении включают только многочлены с целыми коэффициентами. В итоге мы получаем $ \ Phi_n (x) $, поэтому у него есть целые коэффициенты. ■

    Неприводимость круговых многочленов

    Доказать, что многочлен $ \ Phi_n (x) $ всегда неприводим, не так просто, но когда $ n $ простое число, это следует из критерия Эйзенштейна, который легко доказать и который хорошо известен.2 \ nmid b_0c_0 $, поэтому без ограничения общности $ p \ mid b_0, p \ nmid c_0 $. Сравнивая коэффициенты, мы видим, что $ p \ mid b_1c_0 + b_0c_1 $, значит, $ p \ mid b_1 $, и $ p \ mid b_2c_0 + b_1c_1 + b_0c_2 $, так что $ p \ mid b_2 $, и так далее. В конце концов, у нас есть $ p \ mid b_m $, так же как и $ p \ mid a_n $, противоречие. ■

    Теперь мы можем показать случай простых значений $ n $.

    Теорема 4. Когда $ p $ — простое число, $ \ Phi_p (x) $ неприводимо в $ \ mathbb {Z} [x] $.

    Доказательство. Предположим, что $ \ Phi_p (x) = f (x) g (x) $.p $ делятся на $ p $ по малой теореме Ферма (которая может быть доказана очень похожими аргументами). По алгоритму деления мы можем записать $ g_p (x) = f (x) G_p (x) + r_p (x) $, где $ \ deg r_p

    Теперь $ pr_p (\ zeta) = q_p (\ zeta) $ и $ pr_p (x) -q_p (x) $ имеет степень меньше $ d $, поэтому $ pr_p (x) = q_p (x) $ для всех $ x $; в противном случае существовал бы целочисленный многочлен $ f_1 (x) $ меньшей степени, чем $ f (x) $ (как можно меньший) и имеющий $ \ zeta $ в качестве корня, что невозможно, потому что тогда мы могли бы написать $ f ( x) = f_1 (x) s_1 (x) + s_2 (x) $ с $ s_1 (x), s_2 (x) \ in \ mathbb {Z} [x] $ и $ \ deg s_2 <\ deg f_1 $ и $ s_2 (\ zeta) = 0 $, а $ s_2 (x) $ не равно нулю тождественно в силу неприводимости $ f (x).k) = q_ {k \ frac {m} {p}} (\ zeta) $, где коэффициенты $ q_ {k \ frac {m} {p}} $ ограничены одним и тем же $ M $.

    Теперь достаточно показать, что для любого $ 1 \ leq k

    \ [\ begin {eqnarray} m & \ Equiv k \ pmod n \\ m & \ Equiv 1 \ pmod {p_1} \\ &.2 \ Equiv 1 \ pmod n $, и в этих случаях действительно есть такое доказательство. Это теорема Шура и Мурти (Шур доказал положительное утверждение в 1912 году, а Мурти — отрицательное в 1988 году).

    Начнем с результата, говорящего о том, что существование подходящих многочленов влечет за собой бесконечность простых чисел определенных форм.

    Теорема 6. Пусть $ f (x) \ in \ mathbb {Z} [x] $ — многочлен. Тогда множество простых делителей числа $ f (x) $ (то есть множество простых чисел $ q $, которые делят некоторые из этих чисел) бесконечно.

    Доказательство. Сначала предположим, что $ f (0) = \ pm 1 $. Мы имитируем доказательство Евклида для бесконечности простых чисел. Предположим, что $ p_1, …, p_N $ — все простые числа желаемой формы, и пусть $ k $ велико. Рассмотрим число $ f (kp_1 … p_N) $. Поскольку $ k $ велико, это число имеет абсолютное значение больше, чем $ 1 $, поэтому оно делится на некоторый $ p_i $. Однако он удовлетворяет $ f (kp_1 … p_N) \ Equiv f (0) \ Equiv \ pm 1 \ pmod {p_1 …. p_N}, $, поэтому мы пришли к противоречию. Общий случай следует из обозначения $ a = f (0) $ (можно считать $ a \ neq 0 $) и рассмотрения многочлена $ g (x) = \ frac {f (ax)} {a} $, имеющего интеграл коэффициенты и постоянный коэффициент равны $ 1 $.p \ in S_ {n} $, и, кроме того, $ x \ not \ in S_n $. Эти уравнения с дополнительным условием имеют корни $ (p-1) \ varphi (n) $, поэтому корни — это в точности элементы $ S_ {pn} $. Доказательство завершается тем же рассуждением, что и раньше. ■

    Теперь мы можем доказать общее утверждение.

    Теорема 9. Если $ q $ простое число и $ q \ mid \ Phi_n (x) $, то $ q \ mid n $ или $ q \ Equiv 1 \ pmod n $. Следовательно, простых чисел $ q \ Equiv 1 \ pmod n $ бесконечно много.

    Доказательство.{\ alpha_1}} $ и утверждение следует. ■

    Конструируемость правильных многоугольников

    Циклотомические многочлены могут даже применяться для доказательства существования или невозможности определенных конструкций линейки и циркуля. Такие конструкции широко изучались, начиная с классической эпохи, пока алгебра не смогла доказать невозможность некоторых построений. Вопрос, который может быть решен с помощью круговых многочленов, заключается в следующем: «Какие правильные $ n $ -угольники можно построить»? Конструируемость означает, что при наличии линейки (которая может образовывать линию между заданными двумя точками) и циркуля (который может рисовать круг с учетом центра и радиуса) и двух точек на плоскости, можно нарисовать фигуру в виде конечное количество шагов (шаги — это построения из прямых и окружностей).Случаи $ n = 3,4,5 $ были известны еще до Евклида. Эти случаи подразумевают некоторые другие, так как любой угол можно разделить пополам. Гауссу удалось рассмотреть случай $ n = 17 $ и фактически решить всю проблему. Прежде чем приступить к доказательству, нам потребуются некоторые алгебраические понятия.

    Определение. Комплексное число называется алгебраическим , если оно удовлетворяет полиномиальному уравнению с целыми коэффициентами. Если $ \ alpha $ алгебраический, его степень , $ \ text {deg} (\ alpha) $, является наименьшей степенью целочисленного многочлена $ f $, корнем которого он является.Такой многочлен $ f $ (если его коэффициенты не имеют общего множителя) называется минимальным многочленом от $ \ alpha $, обозначаемым $ \ text {minpol} (\ alpha) $.

    Теорема 10. Для любого алгебраического $ \ alpha $ $ \ text {minpol} (\ alpha) $ единственно и неприводимо. Если $ g (\ alpha) = 0 $ и $ g (x) \ in \ mathbb {Z} [x] $, то $ \ text {minpol} (\ alpha) (x) \ mid g (x) $. {a_1}.{a_k} $) является симметричным , если $ f (x_1, …, x_k) = f (x _ {\ sigma (1)}, …, x _ {\ sigma (k)}) $ для любой перестановки $ \ sigma $ из $ \ {1, …, k \} $.

    Определение. Элементарные симметричные многочлены в $ x_1, …, x_k $ — это многочлены $ s_1 = x_1 + … + x_k, s_2 = \ sum_ {i

    Теорема 11. Пусть $ f (x_1, …, x_k) $ симметрично, с коэффициентами из поля $ K $. Тогда $ f (x_1, …, x_k) = P (s_1, …, s_k) $ для некоторого полинома $ P $, коэффициенты которого взяты из $ K $, где $ s_1 ,.n} + 1 $.

    Доказательство. Можно считать, что нам даны точки $ 0 = 0 + 0i $ и $ 1 = 1 + 0i $ на (комплексной) плоскости. На каждом шаге мы строим не более чем квадратное уравнение (полученное путем пересечения прямых или окружностей), коэффициенты которого включают координаты известных точек, и используем его для получения большего количества точек. Мы говорим, что комплексное число $ \ alpha $ можно построить как , если мы можем получить число $ \ alpha $ как точку. Легко видеть, что конструируемые числа образуют поле $ K $, то есть они замкнуты при вычитании и делении (деление можно производить аналогичными треугольниками).{p-2} \ gamma_k \ sqrt [p-1] {P_k (\ eta)} = (p-1) \ zeta. \ end {eqnarray} \]

    Левая часть представляет собой конструктивное число, поскольку $ \ eta $ и $ \ gamma_k $ могут быть построены путем деления углов пополам, $ P_k (\ eta) $ получается из $ \ eta $ с помощью элементарных операций, а корень порядка $ p-1 $ — это просто ряд последовательных квадратных корней. Следовательно, $ \ zeta $ конструктивен, как и $ p $ -угольник. ■

    .

    Добавить комментарий

    Ваш адрес email не будет опубликован. Обязательные поля помечены *